• Shuffle
    Toggle On
    Toggle Off
  • Alphabetize
    Toggle On
    Toggle Off
  • Front First
    Toggle On
    Toggle Off
  • Both Sides
    Toggle On
    Toggle Off
  • Read
    Toggle On
    Toggle Off
Reading...
Front

Card Range To Study

through

image

Play button

image

Play button

image

Progress

1/132

Click to flip

Use LEFT and RIGHT arrow keys to navigate between flashcards;

Use UP and DOWN arrow keys to flip the card;

H to show hint;

A reads text to speech;

132 Cards in this Set

  • Front
  • Back
Following thyroid surgery, it was noted that a patient frequently aspirated fluid into her lungs. Upon examination it was determined that the area of the piriform recess above the vocal fold of the larynx was numb. What nerve may have been injured?

External branch of the superior pharyngeal
Hypoglossal
Internal branch of the superior laryngeal
Lingual
Recurrent laryngeal
The correct answer is: Internal branch of the superior laryngeal

The internal branch of the superior laryngeal is a sensory nerve that pierces the thyrohyoid membrane along with the superior laryngeal artery. It supplies sensory fibers to the mucous membrane of the larynx, superior to the vocal folds. Since this area lost sensation, it appears that the internal branch of the superior laryngeal nerve must have been injured. The external branch of the superior laryngeal nerve is a motor nerve that innervates the cricothyroid muscle--it does not provide any sensory innervation to the larynx. The recurrent laryngeal nerve ascends from the thorax and provides motor innervation to the upper esophagus, lower pharynx, and all the laryngeal muscles except cricothyroideus.

The hypoglossal nerve supplies motor innervation to the muscles of the tongue. The lingual nerve is a sensory nerve for the anterior 2/3 of the tongue. These nerves are not important for innervating the larynx.
The laryngeal muscle most responsible for stretching (elongating) the vocal ligament is the :

Posterior cricoarytenoid
Lateral cricoarytenoid
Thyroarytenoid
Arytenoid
Cricothyroid
The correct answer is: Cricothyroid
Cricothyroid draws the thyroid cartilage forward, tensing the vocal ligaments. This is the one muscle innervated by the external branch of the superior laryngeal nerve. Posterior cricoarytenoid is a very important muscle; it's the only muscle that abducts the vocal folds. This muscle is innervated by the inferior laryngeal nerve, which is a continuation of the recurrent laryngeal nerve. If this muscle is denervated, the vocal folds may be paralyzed in an adducted position, which would prevent air from entering the trachea. Lateral cricoarytenoid, thyroarytenoid, and arytenoid are all muscles that adduct the vocal folds. They are all innervated by the inferior laryngeal nerve.
The muscle most responsible for the abduction of the vocal folds is the :
Arytenoid
Cricothyroid
Lateral cricoarytenoid
Posterior cricoarytenoid
Thyroarytenoid
The correct answer is: Posterior cricoarytenoid
The posterior cricoarytenoid is the only muscle that abducts the vocal folds. This muscle is innervated by the inferior laryngeal nerve, which is a continuation of the recurrent laryngeal nerve. If this muscle is denervated, the vocal folds may be paralyzed in an adducted position, which would prevent air from entering the trachea. Arytenoid, lateral cricoarytenoid, and thyroarytenoid all adduct the vocal folds. Cricothyroid is the only laryngeal muscle innervated by the external branch of the superior laryngeal. It tenses the vocal ligaments by tipping the thyroid cartilage forward relative to the cricoid cartilage.
The nerve that innervates all but one muscle of larynx is the:
Glossopharyngeal nerve, pharyngeal branch
Inferior laryngeal nerve
Superior laryngeal nerve, external branch
Superior laryngeal nerve, internal branch
Thyrohyoid nerve
The correct answer is: Inferior laryngeal nerve
The inferior laryngeal nerve is the continuation of the recurrent laryngeal nerve, superior to the inferior border of the cricoid cartilage. It innervates the posterior cricoarytenoid muscle, which is responsible for abducting the vocal folds. The inferior laryngeal nerve also innervates all of the other intrinsic muscles of the larynx, with the exception of the cricothyroid muscle. The pharyngeal branch of the glossopharyngeal nerve provides sensory innervation to the pharynx, but not to the larynx. The glossopharyngeal nerve only provides motor innervation to stylopharyngeus. The external branch of the superior laryngeal nerve innervates the cricothyroid muscle, which is responsible for stretching the vocal ligaments. The internal branch of the superior laryngeal nerve is responsible for providing sensory innervation to the mucosa of the larynx, superior to the true vocal fold. The thyrohyoid nerve is a branch of ansa cervicalis-- it innervates the thyrohyoid muscle, which elevates the larynx and depresses the hyoid bone (or helps to fix the hyoid in space, so that other muscles, such as tongue muscles, can work).
The vocalis muscle is most responsible for the fine control of phonation because of its attachment into the:
Arytenoid cartilage
Cricoid cartilage
Thyroid cartilage
Vestibular ligament
Vocal ligament
The correct answer is: Vocal ligament
Vocalis represents the innermost fibers of the thyroarytenoid muscle, from the inner surface of the thyroid cartilage. Because vocalis inserts on the vocal ligament, it can relax very specific segments of the vocal ligament to adjust the pitch. Although there are many muscles that insert on the other pieces of cartilage listed, vocalis is the only one that can provide for such fine control of tone because it is inserting right on the vocal ligament.
The constrictor muscles of the pharynx receive their motor nerve supply from the:
Glossopharyngeal nerve
Hypoglossal nerve
Spinal accessory nerve
Sympathetic trunk
Vagus nerve
The correct answer is: Vagus nerve (CN X)
The vagus nerve supplies motor innervation to the muscles of the larynx and pharynx, with the exception of stylopharyngeus (innervated by the glossopharyngeal nerve). It also supplies motor innervation to the palate muscles, with the exception of tensor veli palatini (innervated by the V3 division of the trigeminal nerve). The glossopharyngeal nerve (CN IX) provides the sensory, but not motor, innervation to the pharynx. The hypoglossal nerve (CN XII) provides motor innervation to the muscles of the tongue. The accessory nerve (CN XI) provides motor innervation to the trapezius and the sternocleidomastoid. Finally, the sympathetic trunk supplies sympathetic innervation to the head and neck.
Twenty-four hours following a partial thyroidectomy where the inferior thyroid artery was also ligated (tied off), the patient now spoke with a hoarse voice (whisper), and had difficulty in breathing. Which nerve was injured?

Internal branch of superior laryngeal
Ansa cervicalis
Ansa subclavia
Recurrent laryngeal
External branch of superior laryngeal
The correct answer is: Recurrent laryngeal
The recurrent laryngeal nerve runs with the inferior thyroid artery toward the lower lobes of the thyroid. This means that the recurrent laryngeal nerve would be at risk in any surgery involving the inferior thyroid artery or the inferior poles of the thyroid. The recurrent laryngeal nerve becomes the inferior laryngeal nerve at the inferior border of cricoid cartilage, and this nerve continues on to innervate all the muscles of the larynx with the exception of cricothyroid. So, an injury to the recurrent laryngeal nerve might lead to hoarseness and difficulty breathing (due to a laryngeal spasm).

The internal branch of superior laryngeal runs with the superior laryngeal artery and pierces the thyrohyoid membrane. Ansa cervicalis is a branch of the cervical plexus which hangs in front of the internal jugular vein. It innervates the strap muscles, not the laryngeal muscles. Ansa subclavia is part of the sympathetic trunk which loops around the subclavian artery. Finally, the external branch of the superior laryngeal nerve runs with the superior thyroid artery. This is the artery and nerve that might be damaged when removing the superior lobe of the thyroid.
In performing a thyroidectomy, caution should be exercised when ligating (tying) the inferior thyroid artery, as it lies in a very close relationship to which nerve?
ansa cervicalis
hypoglossal
phrenic
recurrent laryngeal
vagus
The correct answer is: recurrent laryngeal
The recurrent laryngeal nerve crosses the inferior thyroid artery near the lower lobe of the thyroid. This means that the recurrent laryngeal nerve would be at risk in any surgery involving the inferior thyroid artery or the inferior poles of the thyroid. The recurrent laryngeal nerve becomes the inferior laryngeal nerve at the inferior border of cricopharyngeus, and this nerve continues on to innervate all the muscles of the larynx with the exception of cricothyroid. So, you really need to take care to protect the recurrent laryngeal nerve--injuring this structure could lead to hoarseness, permanent loss of voice, or even death due to a laryngeal spasm.

None of the other listed nerves are related to the inferior thyroid artery. Ansa cervicalis is a branch of the cervical plexus which hangs in front of the internal jugular vein. It innervates the strap muscles. The hypoglossal nerve winds behind the internal jugular vein, then sweeps around lateral to the carotid vessels and into the floor of the mouth, where it innervates the tongue muscles. The phrenic nerve lies on top of the anterior scalene muscles--it then travels through the thorax to innervate the diaphragm. Finally, the vagus is found in the carotid sheath--it is associated with the common carotid and the internal carotid arteries.
A 60-year-old man has occasional blackouts and light-headedness. Studies reveal atherosclerotic plaques within the common carotid arteries and the bifurcation of the vessels. A carotid endartectomy is undertaken. Which nerve bundle running vertically within the carotid sheath must the surgeon be careful not to injure?
Accessory
Cervical sympathetic trunk
Glossopharyngeal
Hypoglossal
Vagus
The correct answer is: Vagus
In a carotid endarterectomy, surgeons enter the carotid artery and excise the diseased endothelium and media of the artery so that the lumen is left with a smooth lining. To do this, the surgeons must enter the carotid sheath, which means that any structure within that sheath or near that sheath might be injured. This question specifically asks which structure in the sheath could be injured, so the correct answer is the vagus nerve. The carotid sheath contains 2 nerves: the vagus and the superior ramus of ansa cervicalis. It also contains 3 vessels: internal carotid, common carotid, and internal jugular vein. Any of these structures could be damaged during the procedure.

The accessory nerve is most closely associated with the posterior triangle of the neck. It cuts through this triangle to innervate sternocleidomastoid and trapezius and lies posterior to the carotid vessels The cervical sympathetic trunk lies in front of the prevertebral muscles in the prevertebral fascia. The glossopharyngal nerve comes out of the jugular foramen and divides into 2 branches--pharyngeal and lingual branches. This nerve sweeps near the carotid sheath, but it's not in the carotid sheath. The hypoglossal nerve travels lateral to the carotid vessels before entering the floor of the mouth. It crosses the carotid artery, but it's not in the sheath.
A 55-year-old woman has difficulty swallowing and frequently aspirates fluids while drinking. She is diagnosed as having a skull base tumor occupying the space behind the jugular foramen. Involvement of which structure is responsible for the findings?
Ansa cervicalis
Cervical sympathetic trunk
Accessory nerve
Hypoglossal nerve
Vagus
The correct answer is: Vagus
The vagus nerve, which exits the skull through the jugular foramen, is the motor nerve to the pharynx. So, it allows for swallowing. This patient's symptoms and the location of the tumor clearly point to an injury of the vagus nerve. The accessory nerve also exits the skull through the jugular foramen--it could be injured from this tumor, but the symptoms are not suggestive of an accessory nerve injury. If her accessory nerve was damaged, the patient's sternocleidomastoid and trapezius would be denervated, which means the acromion of her shoulder would sag. The other nerve that comes through the jugular foramen is the glossopharyngeal nerve--this nerve might also be damaged by the tumor.

The other nerves listed do not come from the jugular foramen, so they would not be the first nerves injured by the tumor. Ansa cervicalis is a branch of the cervical plexus that innervates the strap muscles in the anterior neck. The cervical sympathetic trunk is the structure that provides sympathetic innervation to the head and neck. If this trunk was disrupted, the patient would have Horner's syndrome. The symptoms of Horner's syndrome include: ptosis, constricted pupil, lack of sweating on the affected side, flushing on the affected side, and enophthalmos (sunken-in eye). Finally, the hypoglossal nerve exits the skull through the hypoglossal canal, providing motor innervation to the muscles of the tongue. An injury to the hypoglossal nerve would cause the tongue, when protruded, to point to the side of the face with the injured nerve.
While doing a postoperative physical on a patient who has undergone carotid endarterectomy on the right side, it was noted that the tongue deviated toward the right when the patient was asked to point the tongue outward. What nerve crossing the carotid artery must have been injured?
Glossopharyngeal
Hypoglossal
Inferior alveolar
Lingual
Vagus
The correct answer is: Hypoglossal
The hypoglossal nerve crosses the carotid artery before diving into the floor of the mouth, so it might get injured during this procedure. This nerve innervates the muscles of the tongue. When the hypoglossal nerve is injured and a patient sticks out the tongue, the tongue points to the side with the injured nerve. So, this patient, who had the right side endarctectomy, has an injury to the right hypoglossal nerve. The symptoms that you see in this case should point you toward suspecting damage to the hypoglossal nerve.

The glossopharyngeal nerve crosses around the internal carotid artery and the stylopharyngeus muscle--it could theoretically be injured in this procedure, but the symptoms are not consistent with an injury to this structure. The inferior alveolar nerve and lingual nerves are branches of the mandibular division of the trigeminal nerve. They are not associated with the carotid vessels. The vagus nerve is found in the carotid sheath. This nerve can be easily injured by an endarterectomy, although damage to the vagus would cause problems with swallowing, not with tongue movements.
Any manipulation of the superior thyroid artery must be undertaken with care not to damage its small companion nerve, the:
Cervical sympathetic trunk
External branch of the superior laryngeal
Inferior root of the ansa cervicalis
Internal branch of the superior laryngeal
Recurrent laryngeal
The correct answer is: External branch of the superior laryngeal
The external branch of the superior laryngeal nerve runs with the superior thyroid artery--this artery and nerve might be damaged when removing the superior pole of the thyroid. If this nerve was damaged, the cricothyroid muscle would be paralyzed, and a patient would be unable to tense the vocal cords. The internal branch of the superior laryngeal nerve runs with the superior laryngeal artery and provides sensory innervation to the mucous membrane of the larynx, superior to the vocal fold. The recurrent laryngeal nerve crosses the inferior thyroid artery, near the lower pole of the thyroid. This means that the recurrent laryngeal nerve would be at risk in any surgery involving the inferior thyroid artery or the inferior poles of the thyroid. The recurrent laryngeal innervates all the muscles of the larynx with the exception of cricothyroid. So, an injury to the recurrent laryngeal nerve might lead to hoarseness and dificulty breathing.

The cervical sympathetic trunk lies in front of the prevertebral muscles in the prevertebral fascia--it would not be near the structures involved with thyroid surgery. The inferior root of ansa cervicalis is a branch of the cervical plexus. It innervates the strap muscles. Ansa cervicalis hangs in a loop over the carotid vessels; it is not associated with the thyroid gland.
The carotid body is innervated by a branch of the:
Glossopharyngeal nerve
Hypoglossal nerve
Spinal accessory nerve
Sympathetic trunk
Vagus nerve
The correct answer is: Glossopharyngeal nerve (CN IX)
The carotid body is innervated by the carotid branch of the glossopharyngeal nerve. It is a small, reddish-brown ovoid mass that lies on the medial side of the carotid bifurcation, serving as a chemoreceptor that monitors the level of carbon dioxide in the blood. The hypoglossal nerve (CN XII) innervates the muscles of the tongue. The spinal accessory nerve is a motor nerve that innervates the sternocleidomastoid and the trapezius. The sympathetic trunk provides sympathetic innervation to the head and neck. Finally, the vagus nerve supplies motor innervation to the muscles of the pharynx and larynx, with the exception of stylopharyngeus, and the muscles of the palate, with the exception of tensor veli palatini.
A sixty-four-year old man was diagnosed with an acoustic neuroma (tumor of the VIIIth cranial nerve) where it entered the temporal bone. What other cranial nerve might also be affected since this nerve uses the same foramen as the VIIIth in its course?
Abducens
Facial
Glossopharyngeal
Trigeminal
Vagus
The correct answer is: facial
The facial nerve enters the temporal bone with the vestibulocochlear nerve--both cross into the internal acoustic meatus. Abducens (CN VI) crosses through the superior orbital fissure, along with the oculomotor nerve (CN III), the trochlear nerve (CN IV) and the ophthalmic division of the trigeminal nerve (CN V1). The glossopharyngeal (CN IX), vagus (CN X), and spinal accessory nerve (CN XI) all leave through the jugular foramen. Finally, the three divisions of the trigeminal nerve all leave through different foramina: V1, the ophthalmic division, exits through the superior orbital fissure; V2, the maxillary division, leaves through foramen rotundum; V3, the mandibular division, leaves through foramen ovale.
Infections may spread from the nasal cavity to the meninges along the olfactory nerves, as its fibers pass from the mucosa of the nasal cavity to the olfactory bulb via the:
Cribriform plate of the ethmoid
Crista galli
Foramen caecum
Superior orbital fissure
The correct answer is: cribriform plate of the ethmoid.
The olfactory nerve exits the skull through the cribriform plate of the ethmoid bone--an infection in the nasal cavity may be carried to the olfactory bulb by the nerves that are passing through the cribriform plate. The crista galli is a ridge on the ethmoid bone between the two sides of the cribriform plate; it provides an anchor for the falx cerebri. Foramen cecum is a small hole in the frontal bone near the anterior end of the crista galli--it transmits an emissary vein. Finally, the superior orbital fissure is a hole in the sphenoid bone that transmits many cranial nerves: the oculomotor nerve (CN III), the trochlear nerve (CN IV), the ophthalmic division of the trigeminal nerve (CN V1) and the abducens nerve (CN VI) all pass through the superior orbital fissure.
The "danger zone" of the scalp is recognized as which of the following layers?
Galea aponeurotica
Loose connective tissue
Pericranium
Skin
Subcutaneous connective tissue
The correct answer is: loose connective tissue
The scalp is comprised of the following layers, from superficial to deep: Skin, Connective tissue, Aponeurosis, Loose connective tissue, and Pericranium. If you take the first letter of each, it spells SCALP. So, now that you know the order of the layers, you need to figure out which one is the danger zone--the place where infections can spread very quickly. And that layer is layer 4, the loose connective tissue. Pus or blood can spread easily in this layer, and infections in this layer can pass into the cranial cavity through emissary veins. So, infections in the loose connective tissue can pass into intracranial structures such as the brain and meninges. Although layer 2 is a connective tissue layer, too, this layer is a bit thicker and is not a place where infections can easily spread.
A patient who has sustained a fracture to the middle cranial fossa following a fall from a height, might have any of these nerves injured EXCEPT:
Trigeminal
Oculomotor
Abducens
Trochlear
Hypoglossal
The correct answer is: hypoglossal
The middle cranial fossa is the part of the skull that supports the temporal lobes of the brain. It is made of the greater wings of the sphenoid and squamous part of the temporal bones laterally and the petrous part of the temporal bones posteriorly. See Netter Plate 6 and 7 for a better picture of this.

Several cranial nerves enter foramina in the middle cranial fossa; all of these nerves might have been damaged in the fall. The trigeminal nerve (CN V) has three divisions that all leave through spaces in the middle cranial fossa. V1, the ophthalmic division, exits through the superior orbital fissure; V2, the maxillary division, leaves through foramen rotundum; V3, the mandibular division, leaves through foramen ovale. The oculomotor nerve (CN III) crosses through the superior orbital fissure, along with abducens (CN VI), the trochlear nerve (CN IV) and the ophthalmic division of the trigeminal nerve (CN V1). So, all of these nerves might have been damaged in the fall.

The hypoglossal nerve, however, leaves the base of the skull by passing through the hypoglossal canal, which is in the occipital bone and the posterior cranial fossa. It is not likely that this nerve was injured in the fall.
The most likely source of blood in a patient with an epidural hemorrhage is:
Vertebral artery
Middle meningeal artery
Superior cerebral veins
Anterior cerebral artery
Circle of Willis
The correct answer is: middle meningeal artery
The middle meningeal artery supplies most of the dura mater and the bones of the cranial vault. It is this artery or vein that is usually ruptured in an epidural hemorrhage. The vertebral artery carries blood to the deep neck, cervical spinal cord, and hindbrain. It does not supply blood to the dura. Superior cerebral veins drain blood into the superior sagittal sinus. When injured, they bleed into the subdural space and cause a subdural hematoma. The anterior cerebral artery supplies blood to the frontal pole of the brain. It is not found near the epidural space. Finally, the circle of Willis is an important anastomosis at the base of the brain between the following arteries: posterior cerebral arteries, posterior communicating arteries, internal carotid arteries, anterior cerebral arteries, and anterior communicating arteries.
In a fall from a horse, a rider sustains a severe neck injury at the C6 level. In addition to crushing the spinal cord, the left transverse process of the C6 vertebra is fractured. What artery is endangered?
Common carotid
Costocervical
Inferior thyroid
Internal carotid
Vertebral
The correct answer is: Vertebral
The paired vertebral arteries travel through the transverse foramina of the C1-C6 vertebrae. So, since the C6 vertebra was damaged, the vertebral artery could also be ruptured. The other arteries are not closely related with the vertebrae. The common carotid arteries come off the brachiocephalic trunk on the right side and the aortic arch on the left side, giving off many arteries that supply the head and neck. The costocervical trunk is a branch of the subclavian artery that supplies the deep neck and the first 2 intercostal spaces. The inferior thyroid artery is a branch of the thyrocervical trunk that supplies the thyroid. Finally, the internal carotid artery is a branch of the common carotid that joins the circle of Willis and supplies the brain.
A 35-year-old man was admitted to the hospital complaining of double vision (diplopia), inability to see close objects, and blurred vision in the right eye. A vertebrobasilar angiogram revealed an aneurysm of the superior cerebellar artery close to its origin on the right side. The doctor attributed the symptoms to the compression of an adjacent cranial nerve by the aneurysm. The compressed nerve is the:
Abducens (CN VI)
Oculomotor (CN III)
Optic (CN II)
Trigeminal (CN V)
Trochlear (CN IV)
The correct answer is: Oculomotor (CN III)
Given the patient's symptoms, it seems that some nerve involving vision and the ability to control the eye has been injured. Now, you need to think about which nerve might be damaged by an aneurysm of the superior cerebellar artery. The oculomotor nerve, which innervates the superior rectus, medial rectus, inferior rectus, and inferior oblique muscles, passes between the posterior cerebral artery and the superior cerebellar artery. It could be injured if there was enlargement of or damage to either of these vessels. None of the other cranial nerves are in the right position to be injured from an aneurysm of the superior cerebellar artery.
An elderly patient developed fever and worsening headache a few days after sustaining a scalp laceration and subsequent infection due to a car accident. At the hospital the case was diagnosed as meningitis and superior sagittal sinus thrombosis. The attending physician suggested that infection to the sinus initially spread through one of the scalp layers. The scalp layer involved is:
Areolar tissue
Connective tissue
Epicranial aponeurosis
Periosteum
Skin
The correct answer is: Areolar tissue
Areolar tissue is another name for the loose connective tissue layer of the scalp. Pus or blood can spread easily in this layer, and infections in this layer can pass into the cranial cavity through emissary veins. So, infections in the loose connective tissue can pass into intracranial structures such as the superior sagittal sinus, causing conditions like the superior sinus thrombosis.

Remember--the scalp is comprised of the following layers, from superficial to deep: Skin, Connective tissue, Aponeurosis, Loose connective tissue, and Pericranium. (SCALP!) Although layer 2 is a connective tissue layer, too, this layer is thicker and it's not where infections can easily spread.
While riding her bicycle on campus without a helmet a student is hit by a car and falls, hitting her head on the pavement. She is brought to the Emergency Room in an unconscious state with signs of a closed head injury. Tests reveal blood in her cerebrospinal fluid taken from a spinal tap. Diagnosis is of torn cerebral veins as they pass from the brain to the superior sagittal sinus. From which of the following was the bloody fluid taken?
Cavernous sinus
Epidural space
Subarachnoid space
Subdural space
Verterbal venous plexus
The correct answer is: Subarachnoid space
Remember back to the spinal cord - when taking cerebrospinal fluid for a spinal tap, you are removing fluid from the subarachnoid space. The subarachnoid space of the spinal cord is continuous with the subarachnoid space around the brain, so you know that the patient must have had a subarachnoid hemorrhage. Subarachnoid hemorrhages are acute events, often caused by an aneurysm or a closed head injury.

In the spinal cord, the epidural space is filled with fat, and the subdural space is really a potential space only - there is no fluid in this area. The cavernous sinus is a venous sinus of the brain, lateral to the body of the sphenoid bone. The vertebral venous plexus are the veins that drain the spinal cord - they are valveless veins, so they are an important route that cancer cells can use to metastasize.
You have been asked to assess the neurological deficit that might exist in a patient diagnosed with cavernous sinus thrombosis. You will focus your examination on cranial nerves related to the sinus that includes all the following EXCEPT:
Abducens (CN VI)
Facial (CN VII)
Oculomotor (CN III)
Ophthalmic division of the trigeminal nerve (CN V1)
Trochlear (CN IV)
The correct answer is: Facial (CN VII)
The cavernous sinus is a venous sinus of the brain, lateral to the body of the sphenoid bone. All of the cranial nerves and vessels that pass out of the skull at the superior orbital fissure pass through the cavernous sinus. This includes the oculomotor nerve, the trochlear nerve, the ophthalmic division of the trigeminal nerve, and the abducens nerve. The internal carotid artery also passes through the cavernous sinus. Since three of the nerves in the cavernous sinus control the motions of the extraocular muscles, testing eye movements would be a good way to see if nerves in the cavernous sinus were disrupted. The ophthamic division of the trigeminal nerve, which supplies cutaneous sensation to the skin of the upper face, could be tested by evaluating the sensations on the forehead.

The facial nerve is not associated with the cavernous sinus. It passes through the internal acoustic meatus and exits the skull through the stylomastoid foramen. So, you would not need to test to see if the facial nerve was intact.
The glossopharyngeal nerve exits the skull via what opening?
Foramen ovale
Carotid canal
Jugular foramen
Hypoglossal canal
Stylomastoid foramen
The correct answer is: Jugular foramen
The glossopharyngeal nerve (CN IX), vagus (CN X) and accessory nerve (CN XI) exit the skull at the jugular foramen. The posterior meningeal artery enters the skull through this space. The mandibular division of the trigeminal nerve (V3) exits the skull through foramen ovale. The carotid canal is the place where the internal carotid artery and the internal carotid nerve plexus enter the skull. The hypoglossal canal is where the hypoglossal nerve (CN XII) leaves the skull. The stylomastoid foramen is the hole that the facial nerve (CN VII) uses to exit the skull.
An infant was found to have hydrocephalus. Studies revealed that the hydrocephalus was caused because CSF could not get out of the third ventricle. The passage blocked was the:
Central canal
Cerebral aqueduct
Interventricular foramen
Lateral foramen (of Luschka)
Medial foramen (of Magendie)
The correct answer is: Cerebral aqueduct
For CSF to travel from the third ventricle to the 4th ventricle and the central canal of the spinal cord, it must pass through the cerebral aqueduct. So, this is the passageway that must be blocked. The central canal is the space where CSF flows through the spinal cord. It is continuous with the 4th ventricle. The foramen of Luschka and foramen of Magendie are small foramina in the 4th ventricle that allow the CSF to leave the ventricular system and enter the subarachnoid space. The interventricular foramina are passages in the lateral ventricles that allow the CSF to leave the lateral ventricles and enter the 3rd ventricles. See Netter Plate 102 for a diagram of the ventricles of the brain.
A person develops a cavernous sinus thrombosis. Because of its relationship to the sinus, which cranial nerve might be affected?
Abducens
Facial
Mandibular V3
Olfactory
Optic
The correct answer is: Abducens (CN VI)
The cavernous sinus is a venous sinus of the brain, lateral to the body of the sphenoid bone. All of the cranial nerves and vessels that pass out of the skull at the superior orbital fissure pass through the cavernous sinus. This includes the abducens nerve, the oculomotor nerve, the trochlear nerve, and the ophthalmic division of the trigeminal nerve. Any of these nerves might be affected by a cavernous venous sinus thrombosis.

The facial nerve is not associated with the cavernous sinus. It passes through the internal acoustic meatus and exits the skull through the stylomastoid foramen. The mandibular division of the trigeminal nerve (V3) exits the skull through foramen ovale. The olfactory nerves enter the skull through the cribriform plate of the ethmoid bone. The optic nerve exits the skull through the optic canal. None of these nerves are associated with the cavernous sinus.
All of the following nerves exit the cranial cavity by way of bony openings located in the middle cranial fossa EXCEPT:
Abducens
Trochlear
Oculomotor
Trigeminal
Facial
The correct answer is: facial
The middle cranial fossa is the part of the skull that supports the temporal lobes of the brain. It is made of the greater wings of the sphenoid and squamous parts of the temporal bones laterally and the petrous parts of the temporal bones posteriorly. See Netter Plate 6 and 7 for a better picture of this.

Several cranial nerves enter foramina in the middle cranial fossa. The abducens (CN VI) crosses through the superior orbital fissure, along with the oculomotor nerve (CN III), trochlear nerve (CN IV) and ophthalmic division of the trigeminal nerve (CN V1). The trigeminal nerve (CN V) has three divisions that all leave through spaces in the middle cranial fossa. V1, the ophthalmic division, exits through the superior orbital fissure; V2, the maxillary division, leaves through foramen rotundum; V3, the mandibular division, leaves through foramen ovale.

The facial nerve, however, leaves the base of the skull by passing through the internal acoustic meatus, which is in the part of the temporal bone that is in the posterior cranial fossa.
During childbirth, an excessive anteroposterior compression of the head may tear the anterior attachment of the falx cerebri from the tentorium cerebelli. The bleeding that follows is likely to be from which of the following venous sinuses?
Occipital sinus
Sigmoid sinus
Straight sinus
Superior sagittal sinus
Transverse sinus
The correct answer is: Straight sinus
The straight sinus drains the deep cerebrum--it lies within the junction of the falx cerebri and tentorium cerebelli. This is exactly the location that was damaged in birth, so this is the correct answer. The occipital sinus drains the cerebellum--it lies within the dura mater at base of falx cerebelli. It is inferior to the straight sinus. The sigmoid sinus drains the blood from the brain into the internal jugular vein--it lies within sigmoid groove, covered by dura mater. The superior sagittal sinus drains the cerebral hemispheres--it lies superiorly within falx cerebri, near the superior border of the skull. The transverse sinus lies within the attachment of tentorium cerebelli to the inner surface of the calvaria. When trying to understand these sinuses, a picture is really worth a thousand words, so take a look at Netter 97 and 98!
The inferior sagittal sinus is found in the free edge of what structure?
Diaphragma sellae
Falx cerebelli
Falx cerebri
Filum terminale
Tentorium cerebelli
The correct answer is: falx cerebri
The falx cerebri is a crescent-shaped, sagittally-oriented fold of dura mater lying between cerebral hemispheres. The inferior sagittal sinus runs in the inferior margin of the falx cerebri. The falx cerebelli is a small fold of dura mater lying between cerebellar hemispheres--it is the location of the occipital sinus. The diaphragma sellae is a piece of dura mater which forms the roof of the hypophyseal fossa; it is pierced by the stalk of the hypophysis. The tentorium cerebelli is a tent-like sheet of dura mater covering of cerebellum, oriented somewhat transversely. The straight sinus is found in the junction of the falx cerebri and tentorium cerebelli. The filum terminale is an extension of the pia mater below the end of the spinal cord at L2. It is a structure of the spinal cord, not the brain.
Loss of sensation from the temporal region and loss of secretory function of the parotid gland would be caused by interruption of which nerve?
Auriculotemporal
Chorda tympani
Deep temporal, posterior
Facial
Great auricular
The correct answer is: Auriculotemporal nerve
The auriculotemporal nerve is a branch of the mandibular division of the trigeminal nerve (V3). It has two important functions: First, it carries postganglionic parasympathetic fibers to the parotid gland. These fibers come from the otic ganglia, where they synapsed with the presynaptic fibers from the glossopharyngeal nerve (CN IX). Second, the auriculotemporal nerve provides sensory innervation to the skin of anterosuperior ear, part of the external auditory meatus, and the temporomandibular joint. So, the listed symptoms match with an injury to the auriculotemporal nerve.

Chorda tympani is a branch of the facial nerve that provides secretomotor innervation to the submandibular and sublingual glands. It carries preganglionic parasympathetic axons to the submandibular ganglion. In the infratemporal fossa, chorda tympani joins the lingual nerve--it continues with the lingual nerve to the tongue where it supplies taste to the anterior 2/3 of the tongue. The posterior deep temporal nerve is a branch of the mandibular division of the trigeminal nerve which supplies motor innervation to temporalis. The facial nerve (CN VII) innervates all of the muscles of facial expression, and, through the chorda tympani, provides secremotor innervation to the submandibular and sublingual glands as well as taste sensation to the anterior 2/3 of the tongue. Finally, the great auricular nerve comes from the cervical plexus--it provides sensory innervation to the skin of the ear and the skin below the ear.
An elderly man presented with severe pain beneath the left eye, radiating into the lower eyelid, lateral side of the nose and upper lip. What nerve was involved?
Buccal
Infraorbital
Mental
Supratrochlear
Zygomatic
The correct answer is: infraorbital
The infraorbital nerve is a cutaneous nerve from the maxillary division of trigeminal nerve (V2). It innervates the skin of the lateral nose, lower eyelid, upper lip and zygomatic region. This is exactly where this man's pain is, so it seems like his pain must be transmitted on the infraorbital nerve. The buccal branch of the trigeminal nerve is part of the mandibular division (V3)--this nerve provides sensory innervation to the skin of the cheek and the mucosal lining the cheek. It is not a motor nerve--only sensory! (NOTE: The buccal nerve is NOT the same as the buccal branch of the facial nerve. The buccal branch of the facial nerve is a motor nerve only--it innervates several muscles of facial expression. It does not have a sensory component--only motor!) The mental nerve is a branch of the inferior alveolar nerve, which is a branch of the mandibular division of the trigeminal nerve (V3). It provides sensory innervation to the skin of the chin and lower lip. The supratrochlear nerve is a branch of the frontal nerve, from the ophthalmic division of the trigeminal nerve. It gives sensory innervation to the skin of the medial forehead and the medial part of the upper eyelid. The zygomatic nerve is part of the maxillary division of the trigeminal nerve (V2). It provides sensory innervation to the skin of the face lateral and superior to the orbit. For a good picture of these nerves, see Netter Plate 18.
During a face lift operation on a 48-year-old woman, the plastic surgeon inadvertently cut the marginal mandibular branch of the facial nerve. Which of the following muscles would be paralyzed because of the injury?
Buccinator
Depressor anguli oris
Levator anguli oris
Levator labii superioris
Stylohyoid
The correct answer is: depressor anguli oris
The marginal mandibular branch of the facial nerve provides motor innervation to the muscles of facial expression near the lower lip and chin--right where you find depressor anguli oris. So, if the marginal mandibular branch of the facial nerve was injured, depressor anguli oris would be paralyzed. The buccal branches of the facial nerve provide motor innervation to the buccinator muscle and the muscles of the upper lip (levator anguli oris and levator labii superioris). Finally, stylohyoid is innervated by the facial nerve shortly after exiting the stylomastoid foramen--it is not innervated by any of the special branches of the facial nerve that innervate the muscles of facial expression.
As a result of a face lift operation, a 46-year-old woman noticed an asymmetry of the inferior lip and could not fully depress the angle of her mouth on the right side. Which of the following nerves was most likely damaged during the surgery?
zygomatic (VII)
buccal (VII)
mental (V3)
marginal mandibular (VII)
infraorbital (V2)
The correct answer is: marginal mandibular
Depressor anguli oris is the muscle that depresses the angle of the lip--it is innervated by the marginal mandibular branch of the facial nerve. So, if the marginal mandibular branch of the facial nerve was injured, depressor anguli oris would be paralyzed. The zygomatic branches of the facial nerve innervate the muscles of facial expression that are right around the eye, including orbicularis oculi. The buccal branches of the facial nerve innervate the buccinator muscle and other muscles of facial expression that are near the upper lip, like levator anguli oris and levator labii superioris. The mental and infraorbital branches of the trigeminal nerve provide sensory innervation to the skin of the face--they do not innervate any muscles! The mental nerve, a branch of V3 (mandibular division), innervates the skin of the chin and the lower lip. The infraorbital nerve, a branch of V2 (maxillary division), innervates the skin of the lateral nose, lower eyelid, upper lip, and zygomatic region.
An elderly woman complained of a severe pain, felt above the right eye radiating to the upper eyelid, side of the nose and forehead. Branches of which of the following nerves convey pain sensations from areas of the skin described?
maxillary (V2)
greater auricular nerve
ophthalmic (V1)
mandibular (V3)
facial (VII)
The correct answer is: ophthalmic (V1)
The ophthalmic division of the trigeminal nerve provides sensory innervation to the skin of the nose, upper eyelid, and forehead. This is exactly where this woman feels pain, so the ophthalmic division of the trigeminal nerve must be the nerve transmitting the pain. The maxillary division of the trigeminal nerve (V2) provides sensory innervation to the skin of the side of the nose, the cheek, lower eyelid, and upper lip. The mandibular division of the trigeminal nerve (V3) provides sensory innervation to the skin of the chin, lower lip, and lower jaw. The great auricular nerve is a branch of the cervical plexus which supplies sensory innervation to the ear region. Finally, the facial nerve is mostly a motor nerve--it only supplies taste to the anterior 2/3 of the tongue and gives some sensory innervation to the skin of the exernal auditory meatus.
Due to multiple salivary calculi (stones) in the submandibular duct, the submandibular gland of a 45-year-old individual was surgically removed. What major artery directly related to the gland was of special concern to the surgeon?
lingual
superior thyroid
facial
ascending pharyngeal
maxillary
The correct answer is: facial
The facial artery arises from the external carotid and winds toward the inferior border of the mandible, crossing over the submandibular gland. So, if the submandibular gland was removed, the facial artery might be damaged. The lingual artery is a branch of the external carotid that runs in the floor of the mouth. It is associated with the submandibular duct, but not with the gland itself. The superior thyroid artery is a branch of the external carotid which travels anteroinferiorly to supply the upper pole of the thyroid. The ascending pharyngeal artery is a posterior branch of the external carotid which supplies blood to the pharynx. Finally, the maxillary artery is one of the two terminal branches of the external carotid artery--it supplies blood to the maxillary region, muscles of mastication, infratemporal fossa, and deep face. Take a look at Netter Plate 63 to get a better picture of this!
Which nerve provides motor innervation to the buccinator muscle?
Auriculotemporal nerve
Buccal branches of VII
Buccal nerve
Mandibular division of V
Marginal mandibular nerve
The correct answer is: buccal branches of VII
The buccal branches of the facial nerve provide motor innervation to the buccinator muscle. Remember, these buccal branches of the facial nerve are motor nerves only--they do not do any sensory innervation. Don't mix this nerve up with the buccal nerve, which is a branch of the mandibular division of the trigeminal nerve (V3)! The buccal nerve is a sensory nerve only--it does not innervate any muscles; it only gives sensory innervation to the skin of the cheek and the mucosal lining of the cheek. The auriculotemporal nerve is also part of the mandibular division of the trigeminal nerve--it carries the postganglionic parasympathetic fibers to the parotid gland and provides sensory innervation to the skin of the anterosuperior ear, part of the external auditory meatus, and the temporomandibular joint. The marginal mandibular nerve is another branch of the facial nerve--it innervates the muscles of facial expression on the lower lip and chin.
Which nerve carries postganglionic parasympathetic fibers to the parotid gland?
Auriculotemporal nerve
Lesser petrosal nerve
Glossopharyngeal nerve
Great auricular nerve
Marginal mandibular nerve
The correct answer is: Auriculotemporal nerve
The auriculotemporal nerve is a branch of the mandibular division of the trigeminal nerve (V3). It has two important functions: First, it carries postganglionic parasympathetic fibers to the parotid gland. These fibers come from the otic ganglia, where they synapsed with the presynaptic fibers from the glossopharyngeal nerve (CN IX). These presynaptic fibers were carried to the otic ganglia by the lesser petrosal nerve. Second, the auriculotemporal nerve provides sensory innervation to the skin of anterosuperior ear, part of the external auditory meatus, and the temporomandibular joint.

The great auricular nerve is a sensory nerve from the cervical plexus--it innervates the skin of the ear and the skin below the ear. The marginal mandibular nerve is a branch of the facial nerve--it innervates the muscles of facial expression for the lower lip and chin.
A patient is unable to wink; what muscle is affected?
frontalis
levator palpebrae superioris
orbicularis oculi
superior tarsal
zygomaticus major
The correct answer is: orbicularis oculi
Orbicularis oculi is a muscle of facial expression that closes the eyelid for winking. It is innervated by the temporal and zygomatic branches of the facial nerve. Frontalis is the anterior belly of the epicranius muscle; it elevates the eyebrows and wrinkles the forehead. It is innervated by the temporal branches of the facial nerve (VII). Levator palpebrae superioris elevates the upper eyelid; it is innervated by the oculomotor nerve (III). The superior tarsal muscle is another muscle that elevates the eyelid, but it lifts the eyelid invoulntarily. It is innervated by the cervical sympathetic trunk; checking whether this muscle is functioning is a good test to see whether the cervical sympathetic trunk is intact. Finally, zygomaticus major is a muscle at corner of the mouth which elevates and draws the corner of the mouth laterally. It is innervated by the zygomatic and buccal branches of the facial nerve.
What structure lies deepest in the parotid gland?
External carotid artery
External jugular vein
Facial artery
Facial nerve
Retromandibular vein
The correct answer is: external carotid artery
The facial nerve, retromandibular vein, and external carotid artery all course through the parotid gland. From superficial to deep, they are arranged nerve, vein, artery. So, the facial nerve would be the structure most likely to be injured by a superficial injury to the parotid gland, while the external carotid artery is somewhat protected, deep in the gland. The facial artery branches from the external carotid artery before the external carotid artery enters the parotid gland. The external jugular vein is a superficial vein on the lateral surface of the neck.
A deep laceration of the face in the middle of the parotid gland could affect the:
External jugular vein
Facial nerve
Glossopharyngeal nerve
Hypoglossal nerve
Lingual artery
The correct answer is: facial nerve
The facial nerve travels through the parotid gland--it could become injured if there was a deep laceration through the parotid gland. Two other structures found within the parotid gland which might be damaged: the retromandibular vein and the external carotid artery. The nerve is the most superficial structure in the gland. Then, the vein is under the nerve, and the artery is the deepest structure in the gland.

The external jugular vein is a superficial structure on the lateral neck, so it's not really close to the parotid gland. The glossopharyngeal nerve is closely related to the stylopharyngeus muscle--it sweeps along the back of this muscle. It is not related to the parotid gland. The hypoglossal nerve travels laterally to the carotid vessels and then enters the floor of the mouth. This means that it travels inferior to the region of the parotid gland. Finally, the lingual artery is found in the floor of the mouth--far from the parotid gland!
Pain elicited from an infected facial wound is primarily conveyed by what nerve?
Facial
Great auricular
Hypoglossal
Transverse cervical
Trigeminal
The correct answer is: trigeminal
The trigeminal nerve is the nerve that supplies sensory innervation to the skin of the face, so pain sensations will be carried through this nerve. It has 3 divisions. The ophthalmic division (V1) is a sensory nerve that passes through the superior orbital fissure and supplies sensory innervation to the eyeball, conjunctiva, nasal mucosa, medial portion of the nose, upper eyelid, forehead, and scalp. The maxillary division (V2) is a sensory nerve that provides sensory innervation to the cheek, upper lip, lateral portion of nose and lower eyelid. The mandibular division (V3) is a sensory and motor nerve--it supplies the lower lip and chin and the lateral portion of the cheek. V3 also provides motor innervation to the muscles of mastication, tensor veli palatini, mylohyoid, the anterior belly of the digastric, and tensor tympani.

The facial nerve does not provide sensory innervation to the skin of the face--it provides motor innervation to the muscles of facial expression. The great auricular nerve and transverse cervical nerve are branches of the cervical plexus. The great auricular nerve supplies the skin of the ear and the skin below the ear, while the transverse cervical nerve supplies the skin of the anterior neck. The hypoglossal nerve (CN XII) provides motor innervation to the muscles of the tongue.
A 38-year-old female patient complained of parotid pain that increased while eating. Intraoral examination detected some pus oozing from the parotid duct opening. What was the most likely anatomical reference that the physician considered to locate the parotid duct opening?
The correct answer is: Mucosa of the cheek across the 2nd upper (maxillary) molar tooth
The parotid duct opens into the vestibule of the mouth, draining into the mucosa of the cheek near the second upper molar tooth. The duct drains the parotid gland across the masseter and through the cheek. It passes through the buccinator muscle, and pours saliva into the vestibule of the mouth.

The sublingual caruncle is a small bump in the floor of the mouth, near the frenulum of the tongue. This is the site of the opening of the submandibular duct, which drains saliva from the submandibular gland. The mucosa along the sublingual fold contains many openings for the ducts coming from the sublingual gland. See Netter Plate 45 for a picture of all these ducts.
The parotid space contains all EXCEPT:
External carotid artery
Facial nerve
Intraparotid lymph nodes
Medial pterygoid muscle
Retromandibular vein
The correct answer is: Medial pterygoid muscle
The medial pterygoid muscle is not in the parotid space. It serves as the anterior boundary of the parotid fossa. The other structures mentioned are all found within the parotid gland. The facial nerve, retromandibular vein, and external carotid artery all course through the parotid gland. From superficial to deep, they are arranged nerve, vein, artery. The intraparotid lymph nodes are found in the parotid gland also.
Frey's Syndrome is marked by profuse sweating over one cheek, temple, and surrounding areas of the face, precipitated by eating. The condition may be idiopathic, but often follows parotid surgery. The condition is attributable to abberant reinnervation, the redirection of autonomic fibers normally going to salivary glands being redirected to sweat glands. What is the source of the nerve fibers involved?
Facial
Glossopharyngeal
Oculomotor
Trigeminal
Vagus
The correct answer is: Trigeminal
Frey's syndrome is a condition in which the postganglionic parasympathetic nerves that are contained in the auriculotemporal nerve (which normally supply the parotid gland) are redirected toward the sweat glands overlying the parotid gland. This means that a patient with Frey's syndrome sweats in the area over the parotid gland while eating. Since the auriculotemporal nerve is a branch of V3, the nerve fibers involved in Frey's syndrome are from the trigeminal nerve.

The nerve fibers on the external carotid and internal carotid arteries are sympathetic fibers. Remember--sympathetic fibers create periarterial plexuses that travel with the vasculature to reach different targets around the body. However, the parotid gland is innervated parasympathetically, not sympathetically. The glossopharyngeal nerve contributes to the innervation of the parotid gland by supplying preganglionic parasympathetic fibers to the otic ganglia, by way of the lesser petrosal nerve. However, these fibers are not involved with Frey's syndrome--this condition involves the misdirection of the postganglionic parasympathetic fibers. Finally, the vagus is not involved with innervating the parotid gland.
While recovering from multiple dental extractions, an elderly man experienced a radiating pain affecting the lower eyelid, lateral side of the nose, upper lip and over the zygomatic and temporal areas on the left side. Which nerve is involved in the patient's perception of pain?
Facial
Opthalmic division of trigeminal
Glossopharyngeal
Mandibular division of trigeminal
Maxillary division of trigeminal
The correct answer is: Maxillary division of trigeminal
The trigeminal nerve is the nerve that supplies sensory innervation to the skin of the face. It has 3 divisions. The maxillary division of trigeminal (V2) is the one that's important for this case--it is a sensory branch of the trigeminal that provides innervation to the skin of the cheek, upper lip, lower eyelid, and the lateral portion of the nose. This is exactly the area that the patient feels pain, so it is the correct answer. The ophthalmic division (V1) is a sensory nerve that passes through the superior orbital fissure and supplies sensory innervation to the eyeball, conjunctiva, nasal mucosa, medial portion of the nose, upper eyelid, forehead, and scalp. The mandibular division (V3) is a sensory and motor nerve--it supplies skin of the lower lip, chin and lower jaw. V3 also provides motor innervation to the muscles of mastication, tensor veli palatini, mylohyoid, the anterior belly of the digastric, and tensor tympani.

The facial nerve innervates the muscles of facial expression, but it does not provide sensory innervation to the skin of the face. The glossopharyngeal nerve provides sensory innervation to the pharynx and sensory and taste innervation to the posterior 1/3 of the tongue. But, it does not innervate any skin on the face.
To drain an abscess (a closed collection of pus) affecting the cheek area, an emergency room physician used local anesthesia for the surgery. Which of the following nerves must be anesthetized because it carries pain sensation from the cheek area?
buccal (V3)
buccal (VII)
inferior alveolar
lingual
mental
The correct answer is: Buccal (V3)
The buccal nerve is a branch of the mandibular division of the trigeminal nerve that transmits sensory information from the skin of the cheek area and the oral mucosa of the cheek. The buccal nerve is NOT the same as the buccal branches of the facial nerve, which are motor nerves that innervate the buccinator and the facial muscles of the upper lip. The buccal branches of the facial nerve do not have a sensory component!

The inferior alveolar nerve is another branch of V3 that penetrates the mandibular foramen and is the sensory nerve for the mandible and all mandibular teeth. This is the nerve that dentists must anesthesize when they are working on the mandibular teeth. The lingual nerve is also a branch of V3--it transmits general sensation from the tongue. The lingual nerve also receives the chorda tympani, a branch of the facial nerve, which provides the lingual nerve with preganglionic parasympathetic fibers for the submandibular and and sublingual glands, as well as the taste fibers for the anterior 2/3 of the tongue. Finally, the mental nerve is a branch of V3 from the inferior alveolar nerve--it provides sensory innervation to the skin of the chin and lower lip.
A patient who experienced bilateral anterior dislocation of the jaw (temporomandibular joints) could not swallow or talk since the mouth was held open. What is the position of the condyles of the mandible as a result of the dislocation?
against the external acoustic meatus
against the anterior slope of the articular eminence
against the posterior slope of the articular eminence
in the mandibular fossa
in the pterygoid fossa
The correct answer is: against the anterior slope of the articular eminence
The TMJ may dislocate anteriorly due to excessive contraction of the lateral pterygoids. This could happen if someone yawned too much or took a large bite of food. So, you want to look for the answer here that represents anterior movement of the jaw. Answer B, against the anterior slope of the articular eminence, is anterior to the normal position of the mandible, so that's the correct answer.

The external acoustic meatus is posterior to the normal position of the mandible, so you know that the mandible would not touch this structure if it dislocated anteriorly. In its normal position, the head of the mandible is near the posterior slope of the articular eminence and in the mandibular fossa, so these answers are not correct. Finally, the pterygoid fossa (also known as pterygoid fovea) is a fossa found on the mandible itself; it's not a place where the mandible could move to dislocate. For a better understanding of this, see Netter Plate 10 and 11.
In reducing an ankylosis of the TMJ, a surgeon provoked an intense hemorrhage by lacerating the artery coursing transversely just medial to the neck of the condyle. Which artery was involved in the accident?
Buccal
External carotid
Maxillary
Middle meningeal
Superficial temporal
The correct answer is: Maxillary
The maxillary artery is one of the terminal branches of the external carotid artery which is closely related with the TMJ. It travels medial to the neck of the condyle, so it would be the artery damaged in this scenario. The buccal artery is a branch of the maxillary artery that travels in the cheek. It supplies blood to the cheek mucosa and skin. The external carotid artery is the source of the maxillary artery, but it is not the artery lying medial to the neck of the mandibular condyle. The middle meningeal artery is a branch of the maxillary artery which is deep to the lateral pterygoid muscle. It supplies the calvaria and the dura surrounding the brain. Finally, the superficial temporal artery is the other terminal branch of the external carotid artery. It courses posterior and lateral to the head of the mandible to supply the scalp of the lateral side of the head and lateral face. See Netter Plate 35 for a picture of all of these arteries and their interconnections.
The muscle which separates the submandibular triangle from the paralingual space is the:
Digastric, posterior belly
Hyoglossus
Mylohyoid
Stylohyoid
Styloglossus
The correct answer is: temporalis
Temporalis is the one muscle responsible for retracting the mandible--it pulls the mandible backwards. The lateral pterygoid protracts the mandible, or pulls it forward. It is the one jaw muscle that allows for opening the mouth. The masseter is a powerful chewing muscle that elevates the mandible. The medial pterygoid also elevates the mandible; it has a similar position and action to the masseter, but the ramus of the mandible separates the two muscles. Mylohyoid elevates the hyoid bone and the tongue and depresses the mandible.
The predominant muscle most associated with retraction of the mandible is the:
lateral pterygoid
masseter
medial pterygoid
temporalis
mylohyoid
The correct answer is: temporalis
Temporalis is the one muscle responsible for retracting the mandible--it pulls the mandible backwards. The lateral pterygoid protracts the mandible, or pulls it forward. It is the one jaw muscle that allows for opening the mouth. The masseter is a powerful chewing muscle that elevates the mandible. The medial pterygoid also elevates the mandible; it has a similar position and action to the masseter, but the ramus of the mandible separates the two muscles. Mylohyoid elevates the hyoid bone and the tongue and depresses the mandible.
At the temporomandibular joint (TMJ), hinge movements occur between the:
condyle and articular eminence
articular disc and articular eminence
condyle and articular disc
articular disc and articular cavity
condyle and articular cavity
The correct answer is: condyle and articular disc
The TMJ joint is a synovial joint with two articular cavities. Each cavity is responsible for a different movement at the joint. An articular disc sits between the condylar process of the mandible on its inferior side and the mandibular fossa and articular eminence of the temporal bone on the superior side. This disc divides the joint into the two articular cavities, with one cavity acting as a hinge component and the other cavity serving as a gliding component. The lower part of the joint, between the condyle and the articular disc, is the hinge component of the joint. When the joint moves, this hinge component of the joint moves first, to initiate mandibular opening. The upper part of the joint, between the articular disc and the mandibular fossa and articular eminence of the temporal bone, creates the gliding component. During joint movement, this gliding cavity moves after the hinge component to terminate mandibular opening.
A 38-year-old patient complained of acute dental pain. The attending dentist found penetrating dental caries (dental decay) affecting one of the mandibular molar teeth. Which nerve would the dentist need to anesthetize to work on that tooth?
Lingual
Inferior alveolar
Buccal
Mental
Mylohyoid
The correct answer is: Inferior alveolar
The inferior alveolar nerve is a branch of the mandibular division of the trigeminal nerve (V3). It penetrates the mandibular foramen and is the sensory nerve for the mandible and all mandibular teeth. This is the nerve anesthetized by dentists working on the mandibular teeth. It is anesthetized near the mandibular foramen, so sensory nerves from branches distal to that point would be blocked. The lingual nerve is another branch of V3--it transmits general sensation from the tongue. The lingual nerve also receives the chorda tympani, a branch of the facial nerve. The chorda tympani gives the lingual nerve preganglionic parasympathetic fibers for the submandibular and sublingual glands, as well as the taste fibers for the anterior 2/3 of the tongue. The buccal nerve is a branch of V3 that transmits sensory information from the skin of the cheek area and the oral mucosa of the cheek. Remember: The buccal nerve is NOT the same as the buccal branch of the facial nerve, which is a motor nerve innervating the buccinator and muscles of the upper lip!!!

The mental nerve is a branch of V3 from the inferior alveolar nerve--it provides sensory innervation to the skin of the chin and lower lip. The nerve to mylohyoid is also a branch of V3--it innervates the mylohyoid muscle and the anterior belly of the digastric. All of these branches of the inferior alveolar nerve could end up being anesthetized as a result of the dental work, but these nerves are not the dentist's main target!
Incapacity to protrude the mandible indicates a dysfunction of which muscle?
Anterior belly of digastric
Buccinator
Lateral pterygoid
Mylohyoid
Temporalis
The correct answer is: Lateral pterygoid
The lateral pterygoid muscle protrudes the mandible--it pulls the mandible forward to allow for depression of the chin (which is mostly produced by gravity). None of the other muscles help with this function. The anterior belly of the digastric and mylohyoid have similar functions: they both help elevate the hyoid bone and depress the mandible. The buccinator is a muscle in the cheek; it pulls the corner of mouth laterally and presses the cheek against the teeth. Temporalis is important for retracting and elevating the mandible.
Damage to the facial nerve near the stylomastoid foramen would likely cause each of the following motor deficits EXCEPT:
Paralysis of the buccinator muscle
Inability to whistle
Paralysis of the muscles that elevate the mandible
Inability to close the lips
The correct answer is: paralysis of the muscles which elevate the mandible
The masseter, medial pterygoid, and temporalis are all muscles that elevate the mandible. These muscles are innervated by branches of V3, the mandibular division of the trigeminal nerve. So, none of these muscles would be denervated if the facial nerve was injured. The other actions discussed all involve the muscles of facial expression, which are innervated by the facial nerve (CN VII). The buccinator muscle is innervated by the buccal branches of the facial nerve--this muscle pulls the corner of mouth laterally and presses the cheek against the teeth. The orbicularis oris is the muscle that allows for whistling or closing the lips; this muscle is innervated by the buccal branches of the facial nerve.
The lesser petrosal nerve carries preganglionic parasympathetic fibers to the:
Geniculate ganglion
Otic ganglion
Submandibular ganglion
Ciliary ganglion
The correct answer is: otic ganglion
The lesser petrosal nerve is a branch of the glossopharyngeal nerve (CN IX). The lesser petrosal nerve carries presynaptic parasympathetic fibers to the otic ganglion. These fibers synapse in the otic ganglion, and the postsynaptic fibers travel on the auriculotemporal nerve to innervate the parotid gland. The geniculate ganglion is the sensory ganglion of the facial nerve--it holds the cell bodies of the neurons that carry taste sensations from the anterior 2/3 of the tongue. The submandibular ganglion receives preganglionic parasympathetic fibers from the chorda tympani; the postsynaptic fibers from the submandibular ganglion go to the sublingual and submandibular glands. Finally, the ciliary ganglion receives preganglionic parasympathetic fibers from the inferior division of the oculomotor nerve. The postganglionic fibers from this ganglion leave via short ciliary nerves and innervate sphincter pupillae and the ciliary muscle of the eye. Also remember that sensory and sympathetic fibers are passing through the ciliary ganglion and distributing on the short ciliary nerves, but these fibers never synapse in that ganglion.
What bony feature of the mandible can be used to find and palpate the facial artery?
Oblique line
Mental trigone
Angle
Premasseteric notch
The correct answer is: Premasseteric notch
The facial artery arises from the external carotid artery and winds around the inferior border of the mandible deep to the platysma, immediately anterior to the insertion of masseter muscle. This is the area of the premasseteric notch. The notch lies anterior to the angle of the mandible, so angle would not be a landmark to find the facial artery.
Which of the following suprahyoid muscles would be paralyzed if the inferior alveolar nerve were severed at its origin?
Geniohyoid m.
Hyoglossus m.
Mylohyoid m.
Stylohyoid m.
The correct answer is: Mylohyoid
Mylohyoid is innervated by the nerve to mylohyoid, which is a branch of the inferior alveolar nerve. The inferior alveolar nerve is a branch of the mandibular division of the trigeminal nerve. Geniohyoid is a muscle that spans from the mental spines of the mandible to the body of the hyoid bone--it elevates the hyoid and depresses the mandible. It is innervated by the ventral primary ramus of spinal nerve C1 via fibers carried by the hypoglossal nerve. Hyoglossus is a tongue muscle; it is innervated by the hypoglossal nerve (CN XII). Finally, stylohyoid is a muscle that spans from the styloid process to the hyoid bone. It elevates and retracts the hyoid, and it is innervated by the facial nerve.
Which nerve is endangered during surgical removal of an impacted third mandibular molar tooth?
Hypoglossal n.
Glossopharyngeal n.
Inferior alveolar n.
Lingual n.
The correct answer is: inferior alveolar nerve
The inferior alveolar nerve provides sensory innervation to the mandibular teeth. This nerve runs in the mandibular foramen, near the roots of the teeth. So, it might be endangered when removing an impacted tooth. The hypoglossal nerve travels lateral to the carotid vessels and through the floor of the mouth. The glossopharyngeal nerve consists of pharyngeal branches that provide sensory innervation to the upper pharynx and lingual branches that sweep around the stylopharyngeus muscle and continue to the base of the tongue. Finally, the lingual nerve, which is from the mandibular division of the trigeminal nerve, is found in the floor of the mouth. These three nerves are far from the area of the impacted tooth, so they would not be endangered by the dental procedure.
After the mandibular condyle is moved forward onto the articular eminence (e.g., by opening the mouth widely), what muscle can then retract the mandible?
Superficial head of masseter m.
Deep head of masseter m.
Posterior part of temporalis m.
Anterior part of temporalis m.
The correct answer is: posterior part of temporalis
The fibers of the posterior part of temporalis retract the mandible; the fibers from the anterior part of temporalis elevate the mandible. The anterior and deep heads of the masseter muscle are both important for elevating the mandible--remember, the masseter is the very powerful chewing muscle!
Two nerves usually emerge from between the two heads of the lateral pterygoid muscle: the anterior deep temporal nerve and the:
Masseteric n.
Buccal n.
Lingual n.
Inferior alveolar n.
The correct answer is: buccal
The lateral pterygoid muscle has 2 heads: The superior head of the muscle inserts into the disc and capsule of the temporomandibular joint while the inferior head inserts into the neck of the mandible (the pterygoid fovea). There are two nerves coming between the heads of the lateral pterygoid muscle: the anterior deep temporal nerve and the buccal nerve. See Netter Plate 42A for a picture of these nerves. The masseteric nerve courses superior to the 2 heads of the lateral pterygoid, not between the 2 heads. The lingual nerve travels inferior to the two heads of the muscle. The inferior alveolar nerve also travels inferior to the two heads of the lateral pterygoid before it enters the mandibular foramen.
The chorda tympani enters the infratemporal fossa after it exits the:
Stylomastoid foramen
Foramen spinosum
Foramen lacerum
Petrotympanic fissure
The correct answer is: petrotympanic fissure
The chorda tympani exits the skull through the petrotympanic fissure. The stylomastoid foramen is the space that the facial nerve travels through to leave the skull--remember, the chorda tympani has already separated away from the facial nerve by this point. Foramen spinosum is a hole in the base of the skull that transmits the middle meningeal artery and vein. Finally, foramen lacerum is a ragged foramen that is an artifact of a dried skull. In life, it is closed by cartilage. Nothing passes directly through foramen lacerum, although both greater and deep petrosal nerve pass through some of the cartilage that fills it, in order to enter the pterygoid canal at the anterior margin of foramen lacerum.
Paralysis of which of the following muscles would impede retraction of the mandible?
Buccinator
Lateral pterygoid, lower portion
Lateral pterygoid, upper (sphenomeniscus) portion
Medial pterygoid
Temporalis
The correct answer is: Temporalis
Temporalis is the important muscle for retracting the mandible! Buccinator is a muscle on the side of the face which pulls the corner of mouth laterally and presses the cheek against the teeth. Both portions of the lateral pterygoid protract the mandible and open the mandible. (Remember: lateral pterygoid is the only muscle that opens the mandible!) Finally, the medial pterygoid protracts and elevates the mandible.
A cranial fracture through the foramen ovale that compresses the enclosed nerve, will have an effect on all muscles EXCEPT :
Tensor tympani
Masseter
Buccinator
Mylohyoid
Temporalis
The correct answer is: buccinator
The mandibular division of the trigeminal nerve (V3) is transmitted through foramen ovale. Branches from this nerve innervate the muscles of mastication, like temporalis and masseter. A branch from V3 also innervates tensor tympani, which is a muscle that dampens the vibrations of the tympanic membrane. Mylohyoid is innervated by a branch of the inferior alveolar nerve which is also from V3--this muscle elevates the hyoid bone and the tongue and depresses the mandible.

Buccinator is a muscle on the side of the cheek which is innervated by the facial nerve (VII). This muscle, which pulls the corner of mouth laterally and presses the cheek against the teeth, would still be functional even after disrupting the mandibular division of the trigeminal nerve.
Which muscle is also known as the sphenomeniscus?
Inferior head of the lateral pterygoid
Masseter
Medial pterygoid
Superior head of the lateral pterygoid
Temporalis
The correct answer is: Superior head of the lateral pterygoid
The superior head of lateral pterygoid is sometimes called sphenomeniscus due to its insertion into the disc of the temporomandibular joint. Remember, the superior head of the lateral pterygoid inserts into the disc of the TMJ, while the inferior head of the lateral pterygoid inserts into the neck of the mandible (pterygoid fovea). None of the other listed muscles insert into a joint like the superior head of the lateral pterygoid.
Forward movement of the condyle of the mandible during wide opening of the jaws is accomplished mainly by the:
anterior part of temporalis muscle
lateral pterygoid muscle
masseter muscle
medial pterygoid muscle
posterior part of the temporalis muscle
The correct answer is: lateral pterygoid muscle
Remember--the lateral pterygoid muscle is the one muscle of mastication that is responsible for opening the jaw! It also draws the mandible forward. The anterior part of the temporalis muscle elevates the mandible, while the posterior part of temporalis retracts the mandible. Masseter is a powerful chewing muscle that elevates the mandible. The medial pterygoid muscle elevates the mandible and can help to move the mandible forward. But, when the mouth is open, the lateral pterygoid will be the most important muscle for drawing the mandible forward.
To drill a mandibular tooth without causing undue pain, a dentist has injected an anesthetic into the space located between the medial pterygoid muscle and the mandible near the lingula. Given the nerves passing through the immediate vicinity of the injection site, where would one expect anesthesia in addition to the mandibular teeth?
back of tongue
external ear
maxillary incisor teeth
skin of chin
upper lip
The correct answer is: Skin of chin
The dentist would need to numb the inferior alveolar nerve, which is the nerve that runs in the mandibular foramen and provides sensory innervation to the teeth. The inferior alveolar nerve also gives off the mental nerve, which is a sensory nerve innervating the skin of the chin. So, if the inferior alveolar nerve was anesthetized, the mental nerve would be anesthetized, too. Then, the patient would lose sensory innervation to the skin of the chin.

The back of the tongue (posterior 1/3) receives sensory innervation from the glossopharyngeal nerve. This nerve would not be exposed to the anesthetic. The external ear is innervated by the auriculotemporal nerve, which is not near the area of anesthesia. The maxillary incisor teeth are innervated by the superior alveolar nerves, which are branches of the maxillary division of the trigeminal nerve (V2). The skin of the upper lip is also innervated by a branch of V2--the infraorbital nerve. These branches of V2 would not be exposed to the anesthesia.

If too much anesthesic was injected or if the parotid fascia was pierced by the needle, it would be possible for the anesthesic to diffuse through the soft tissue and paralyze the facial nerve. A high dose of anesthesic might also diffuse to paralyze the muscles of mastication.
The middle meningeal artery:
enters the skull through the foramen ovale
passes through a split in the trunk of the mandibular nerve (V3)
is typically a branch of the second part of the maxillary artery
supplies blood to the temporal lobe of the brain
usually arises deep to the neck of the mandible
The correct answer is: usually arises deep to the neck of the mandible
The middle meningeal artery is always found deep to the lateral pterygoid muscle, so it is arising deep to the neck of the mandible. It enters the skull by passing through the foramen spinosum, not through foramen ovale. Remember--foramen ovale transmits the mandibular branch of trigeminal. Although the middle meningeal artery appears to pass through a fork in the auriculotemporal nerve, it does not pass through a split in the trunk of V3. The middle meningeal artery is not a branch of the second part of the maxillary artery--it is a very early branch that separates from the maxillary artery almost immediately. It supplies blood to the dura mater and the bones of the cranial vault, but it does not supply the temporal lobe of the brain. The temporal lobe is supplied by the middle cerebral artery.
The surgical removal of a metastatic tumor in the infratemporal fossa caused an intense hemorrhage. The surgeon clamped the main source of arterial supply to the area, which is the:
Internal carotid
Lingual
Maxillary
Posterior auricular
Superficial temporal
The correct answer is: Maxillary artery
The maxillary artery is one of the terminal branches of the external carotid artery. It enters the infratemporal fossa and is the major source of blood for that region. The internal carotid artery is the primary blood supply to the brain. The lingual artery is another branch of the external carotid artery--it supplies blood to the tongue and the floor of the mouth. The posterior auricular is a branch of the external carotid artery that supplies the external ear, scalp and the deeper structures posterior to the ear. The superficial temporal artery is the other terminal branch of the external carotid artery -- it supplies blood to the scalp of the lateral side of the head and the lateral face.
In acute inflammation (arthritis) of the TMJ, the muscle most likely to be affected by the inflammatory process is the:
Temporal
Medial pterygoid
Masseter
Lateral pterygoid
The correct answer is: lateral pterygoid
The inferior head of the lateral pterygoid inserts into the neck of the mandible, while the superior head of the lateral pterygoid inserts directly into the capsule and articular disk of the temporomandibular joint. So, this muscle would be affected by arthritis of the joint. This muscle is the only one that opens the mandible, so this movement might be weakened if there was inflammation at the TMJ.

Temporalis elevates and retracts the mandible; it inserts on the coronoid process of the mandible and the anterior surface of the ramus of the mandible. The medial pterygoid muscle protracts and elevates the mandible; it inserts on the medial surface of the ramus and angle of the mandible. Finally, the masseter is the muscle that powerfully elevates the mandible--it inserts on the lower half of the ramus of the mandible. None of these other muscles are inserting into the TMJ, so they would not be impaired quite as much as the lateral pterygoid.
In explaining the pain caused by the drilling of a mandibular molar tooth crown to a freshman medical student, a dentist identified the nerve conducting the pain sensations as the:
Lingual
Mylohyoid
Inferior alveolar
Buccal
The correct answer is: Inferior alveolar
The inferior alveolar nerve is a branch of the mandibular division of the trigeminal nerve (V3). It travels through the mandibular foramen and provides sensory innervation to the mandibular teeth. The lingual nerve is another branch of V3--it travels in the floor of the mouth and provides sensory innervation to the anterior 2/3 of the tongue and the floor of the mouth. The nerve to mylohyoid is a motor branch of V3--it provides motor innervation to mylohyoid and the anterior belly of the digastric. Finally, the buccal nerve is a sensory branch of V3 that supplies the cheek and oral mucosa.
The temporomandibular joint is characterized by all EXCEPT:
A capsule strengthened by ligaments on its lateral side only
A completely flat surface for its gliding action
An articular disc
Extracapsular ligaments
Two joint cavities of different shapes
The correct answer is: A completely flat surface for its gliding action
The TMJ joint is a synovial joint with two articular cavities. Each cavity is responsible for a different movement at the joint. The lower part of the joint is the hinge component of the joint. When the joint moves, this hinge component of the joint initiates mandibular opening. The upper part of the joint is the gliding component. During joint movement, this gliding cavity moves to terminate mandibular opening. The gliding cavity is the space between the articular disc and the mandibular fossa and articular eminence of the temporal bone--it's not a completely flat surface.

There are extracapsular ligaments around the TMJ joint capsule, but these ligaments are on the lateral side only. The lateral ligament reinforces the lateral part of the capsule, while other ligaments (the stylomandibular and sphenomandibular) only have a minor role in stabilizing the joint capsule. There is also an articular disc dividing the two components of the joint, and the two cavities are different shapes. See Netter Plate 11 for a picture of all of these structures.
In dislocation of the jaw, displacement of the articular disc beyond the articular tubercle of the temporomandibular joint results from excessive contraction of which muscle?
Buccinator
Lateral pterygoid
Medial pterygoid
Masseter
Temporalis
The correct answer is: Lateral pterygoids
The TMJ may dislocate anteriorly due to excessive contraction of the lateral pterygoids (for example, during excessive opening of the mouth). This could happen if someone yawned too much or took a large bite of food. Posterior dislocations of the TMJ are rare due to resistance from the postglenoid tubercle and the strong lateral ligaments.

The buccinator is an important muscle for mastication because it keeps the cheek taut, so the cheek can press against the molars. However, it is a facial muscle innervated by the facial nerve, and it is not active at the TMJ. The medial pterygoid, masseter and temporalis produce motion at the TMJ, but contracting these muscles does not cause the joint to dislocate.
Sympathetic fibers reach the tongue by way of the:
lingual nerve
maxillary artery
hypoglossal nerve
lingual artery
glossopharyngeal nerve
The correct answer is: Lingual artery
Remember--all over the body, the vasculature is the number one target of sympathetic nerves. Sympathetic nerves help to constrict the vasculature, and they are found covering arteries in periarterial plexuses. In the head, sympathetics travel to targets on vessels. So, you know that the sympathetic nerves are coming to the tongue on an artery--now, you just need to determine what artery goes to the tongue. And that artery is the lingual artery.

The lingual artery is the second branch off the anterior side of the external carotid artery. It travels in the floor of the mouth and supplies blood to the tongue, suprahyoid muscles, and the palatine tonsil. This is the artery going to the targeted area, so that's the answer you're looking for. The maxillary artery is one of the 2 terminal branches of the external carotid artery. It supplies blood to the deep face and infratemporal fossa. It is not going to the tongue.

As far as the nerves go... The lingual nerve is a branch of V3, the mandibular division of the trigeminal nerve. It supplies general sensation (touch and temperature) to the anterior 2/3 of the tongue. (Taste sensation for the anterior 2/3 of the tongue arrives at the tongue by way of the lingual nerve, but the original fibers for taste came from the chorda tympani, a branch of CN VII.) The hypoglossal nerve (CN XII) provides motor innervation to the muscles of the tongue. The glossopharyngeal nerve (CN IX) provides taste sensation and general sensation to the posterior 1/3 of the tongue.
After a radiograph revealed a sialolith (stone) in a patient's right submandibular duct, the surgeon exposed the duct via an intraoral approach. In this approach, what tissues or structures must be cut through?
Mucous membrane only
Mucous membrane and genioglossus muscle
Mucous membrane and mylohyoid muscle
Mucous membrane and hyoglossus muscle
The correct answer is: Mucous membrane only
The submandibular duct is found deep to the mucous membrane of the mouth, but superficial to the muscles of the tongue and the muscles on the floor of the mouth. So, genioglossus, mylohyoid, and hyoglossus are deep to the submandibular duct. See Netter Plate 56B for a picture of these relationships.
Damage of the lingual nerve before it is joined by the chorda tympani in the infratemporal fossa would cause loss of:
general sensation to the anterior two thirds of the tongue
general sensation to the posterior one third of the tongue
secretion of the submandibular gland
taste sensation from the anterior two thirds of the tongue
taste sensation from the posterior one third of the tongue
The correct answer is: General sensation to the anterior 2/3 of the tongue
The lingual nerve is a branch of V3, the mandibular division of the trigeminal nerve. It transmits general sensation from the anterior 2/3 of the tongue. Damaging this nerve anywhere along its course would cause someone to lose general sensation to the anterior 2/3 of the tongue. The chorda tympani is a branch of VII--it carries taste fibers to the anterior 2/3 of the tongue and presynaptic parasympathetic fibers to the submandibular ganglion. The fibers from the chorda tympani join the lingual nerve as they travel to the submandibular ganglion and the anterior tongue. However, if the lingual nerve was damaged before the chorda tympani had joined it, the chorda tympani fibers would still be intact. So, there would still be taste innervation to the anterior 2/3 of the tongue and the submandibular gland would still secrete. If the lingual nerve was disrupted after the chorda tympani had already joined it, there would be no secretomotor innervation to the submandibular and sublingual glands, and there would be no taste or general sensation to the anterior 2/3 of the tongue.

As for the posterior 1/3 of the tongue, it receives taste and general sensation from the glossopharyngeal nerve (CN IX).
The teeth and gums separate the oral cavity proper from the:
Nasal cavity
Oral vestibule
Oropharynx
Paralingual space
Submandibular space
The correct answer is: oral vestibule
The oral vestibule is the space in the mouth lying between the lips and the teeth. See Netter Plate 45 for an illustration of the mouth--although the vestibule isn't labeled, you can get some idea of where it lies. The nasal cavities are spaces that connect the nasopharynx with the external environment. The roof of the nasal cavity is made of the cribriform plate of the ethmoid bone. The floor of the nasal cavity is made of the hard palate. The medial wall is made by the nasal septum, and the lateral wall is the place where the inferior, superior, and middle conchae project into the nasal cavity.

The oropharynx is the middle part of the pharynx, found behind the mouth. It communicates anteriorly with oral cavity through the palatoglossal arch, superiorly with the nasopharynx through the posterior margin of soft palate, and inferiorly with the laryngopharynx at the superior margin of epiglottis. The oropharynx contains the palatine tonsil, which is located between the palatoglossal arch and the palatopharyngeal arch.

The paralingual space is a space inside the floor of the mouth. It is bounded by the mylohyoid muscle, the lateral tongue, the hyoid bone, and the oral mucosa. The paralingual space contains the deep portion of the submandibular gland, the lingual nerve and submandibular ganglia, the sublingual gland and fold, and the hypoglossal nerve.

Finally, the submandibular space is a space defined by the body of the mandible and the anterior and posterior digastric muscles. It contains the superficial submandibular gland, mylohyoid and stylohyoid muscles, the facial artery, and the facial vein.
Description for the following questions: Examination of a patient with an ulcerative carcinoma of the posterior third of the tongue revealed bleeding from the lesion and difficulty swallowing (dysphagia).
The bleeding was seen to be arterial; which of the following arteries was involved?
Deep lingual
Dorsal lingual
Facial
Sublingual
Tonsillar
The correct answer is: Dorsal lingual artery
The dorsal lingual artery runs on the superficial surface of the tongue--it is a branch of the lingual artery that delivers blood to the posterior superficial tongue. So, this artery must be the source of the hemorrhage. The deep lingual artery and sublingual artery are two terminal branches of the lingual artery. These branches run in the floor of the mouth (sublingual) and the deep surface of the tongue (deep lingual). See Netter Plate 53 for a picture. The facial artery is a branch of the external carotid artery that courses across the face. The tonsillar artery is a branch of the facial artery that supplies blood to the palatine tonsil.
Description for the following questions: Examination of a patient with an ulcerative carcinoma of the posterior third of the tongue revealed bleeding from the lesion and difficulty swallowing (dysphagia).

The difficulty in swallowing was due to involvement of which muscle that elevates the tongue?
Genioglossus
Hyoglossus
Styloglossus
Stylohyoid
Stylopharyngeus
The correct answer is: Styloglossus
Styloglossus retracts and elevates the tongue. Genioglossus is a large, fan shaped muscle with many actions: its inferior fibers protrude the tongue, its middle fibers depress the tongue, and its superior fibers draw the tip back and down. Hyoglossus retracts and depresses the tongue. All of these muscles are innervated by the hypoglossal nerve (CN XII). Stylohyoid elevates and retracts the hyoid bone. It is innervated by the facial nerve (CN VII). Stylopharyngeus elevates the larynx--it is innervated by the glossopharyngeal nerve (IX).
Description for the following questions: Examination of a patient with an ulcerative carcinoma of the posterior third of the tongue revealed bleeding from the lesion and difficulty swallowing (dysphagia).

Cutting of the hypoglossal nerve in the hypoglossal canal would not interrupt the nerve supply to the:
Hyoglossus muscle
Genioglossus muscle
Palatoglossus muscle
Styloglossus muscle
The correct answer is: Palatoglossus muscle
All of the tongue muscles are innervated by the hypoglossal nerve. So, hyoglossus, genioglossus, and styloglossus are all innervated by the hypoglossal nerve. Although palatoglossus sounds like a tongue muscle, it is a palatal muscle, innervated by the vagus nerve. (Palatoglossus is the only "glossus" muscle that is not innervated by the hypoglossal nerve.)
Description for the following questions: Examination of a patient with an ulcerative carcinoma of the posterior third of the tongue revealed bleeding from the lesion and difficulty swallowing (dysphagia).

The contents of the paralingual space do NOT include the:
Hypoglossal nerve
Lingual artery
Lingual nerve
Submandibular gland
Sublingual gland
The correct answer is: Lingual artery
The paralingual space is a space inside the floor of the mouth. It is bounded by the mylohyoid muscle, the lateral tongue, the hyoid bone, and the oral mucosa. The paralingual space contains the deep portion of the submandibular gland, the lingual nerve and submandibular ganglion, the sublingual gland and fold, and the hypoglossal nerve. The lingual artery is not in the paralingual space.
A patient is unable to taste a piece of sugar placed on the anterior part of the tongue. Which cranial nerve is most likely to have a lesion?
Facial nerve
Glossopharyngeal nerve
Hypoglossal nerve
Trigeminal nerve
Vagus nerve
The correct answer is: facial nerve
Taste fibers to the anterior 2/3 of the tongue come from the chorda tympani--a branch of the facial nerve. So, if the patient has lost taste sensation to the anterior 2/3 of the tongue, the patient might have a lesion of the facial nerve. The glossopharyngeal nerve (CN IX) carries taste and general sensory fibers to the posterior 1/3 of the tongue. The hypoglossal nerve (CN XII) gives motor innervation to all the muscles of the tongue (ie, all the muscles that end in glossus except for palatoglossus, which is a palate muscle innervated by the vagus). The trigeminal nerve supplies general sensory fibers to the anterior 2/3 of the tongue via the lingual nerve, a branch of V3. Remember, the chorda tympani fibers jump on the lingual nerve to reach the tongue, but the chorda tympani fibers originate from the facial nerve! Finally, the vagus nerve supplies general and taste sensation to a very small part of the posterior tongue, right next to the epiglottis.
Description for the following questions: Examination of a patient with an ulcerative carcinoma of the posterior third of the tongue revealed bleeding from the lesion and difficulty swallowing (dysphagia).
The bleeding was seen to be arterial; which of the following arteries was involved?
Deep lingual
Dorsal lingual
Facial
Sublingual
Tonsillar
The correct answer is: Dorsal lingual artery
The dorsal lingual artery runs on the superficial surface of the tongue--it is a branch of the lingual artery that delivers blood to the posterior superficial tongue. So, this artery must be the source of the hemorrhage. The deep lingual artery and sublingual artery are two terminal branches of the lingual artery. These branches run in the floor of the mouth (sublingual) and the deep surface of the tongue (deep lingual). See Netter Plate 53 for a picture. The facial artery is a branch of the external carotid artery that courses across the face. The tonsillar artery is a branch of the facial artery that supplies blood to the palatine tonsil.
Description for the following questions: Examination of a patient with an ulcerative carcinoma of the posterior third of the tongue revealed bleeding from the lesion and difficulty swallowing (dysphagia).

The difficulty in swallowing was due to involvement of which muscle that elevates the tongue?
Genioglossus
Hyoglossus
Styloglossus
Stylohyoid
Stylopharyngeus
The correct answer is: Styloglossus
Styloglossus retracts and elevates the tongue. Genioglossus is a large, fan shaped muscle with many actions: its inferior fibers protrude the tongue, its middle fibers depress the tongue, and its superior fibers draw the tip back and down. Hyoglossus retracts and depresses the tongue. All of these muscles are innervated by the hypoglossal nerve (CN XII). Stylohyoid elevates and retracts the hyoid bone. It is innervated by the facial nerve (CN VII). Stylopharyngeus elevates the larynx--it is innervated by the glossopharyngeal nerve (IX).
Description for the following questions: Examination of a patient with an ulcerative carcinoma of the posterior third of the tongue revealed bleeding from the lesion and difficulty swallowing (dysphagia).

Cutting of the hypoglossal nerve in the hypoglossal canal would not interrupt the nerve supply to the:
Hyoglossus muscle
Genioglossus muscle
Palatoglossus muscle
Styloglossus muscle
The correct answer is: Palatoglossus muscle
All of the tongue muscles are innervated by the hypoglossal nerve. So, hyoglossus, genioglossus, and styloglossus are all innervated by the hypoglossal nerve. Although palatoglossus sounds like a tongue muscle, it is a palatal muscle, innervated by the vagus nerve. (Palatoglossus is the only "glossus" muscle that is not innervated by the hypoglossal nerve.)
Description for the following questions: Examination of a patient with an ulcerative carcinoma of the posterior third of the tongue revealed bleeding from the lesion and difficulty swallowing (dysphagia).

The contents of the paralingual space do NOT include the:
Hypoglossal nerve
Lingual artery
Lingual nerve
Submandibular gland
Sublingual gland
The correct answer is: Lingual artery
The paralingual space is a space inside the floor of the mouth. It is bounded by the mylohyoid muscle, the lateral tongue, the hyoid bone, and the oral mucosa. The paralingual space contains the deep portion of the submandibular gland, the lingual nerve and submandibular ganglion, the sublingual gland and fold, and the hypoglossal nerve. The lingual artery is not in the paralingual space.
A patient is unable to taste a piece of sugar placed on the anterior part of the tongue. Which cranial nerve is most likely to have a lesion?
Facial nerve
Glossopharyngeal nerve
Hypoglossal nerve
Trigeminal nerve
Vagus nerve
The correct answer is: facial nerve
Taste fibers to the anterior 2/3 of the tongue come from the chorda tympani--a branch of the facial nerve. So, if the patient has lost taste sensation to the anterior 2/3 of the tongue, the patient might have a lesion of the facial nerve. The glossopharyngeal nerve (CN IX) carries taste and general sensory fibers to the posterior 1/3 of the tongue. The hypoglossal nerve (CN XII) gives motor innervation to all the muscles of the tongue (ie, all the muscles that end in glossus except for palatoglossus, which is a palate muscle innervated by the vagus). The trigeminal nerve supplies general sensory fibers to the anterior 2/3 of the tongue via the lingual nerve, a branch of V3. Remember, the chorda tympani fibers jump on the lingual nerve to reach the tongue, but the chorda tympani fibers originate from the facial nerve! Finally, the vagus nerve supplies general and taste sensation to a very small part of the posterior tongue, right next to the epiglottis.
The chorda tympani contains which component before it joins the lingual nerve?
Preganglionic sympathetics
Postganglionic sympathetics
Preganglionic parasympathetics
Postganglionic parasympathetics
Taste fibers to the posterior third of the tongue
The correct answer is: preganglionic parasympathetics
The chorda tympani contains preganglionic parasympathetic nerves that go to the submandibular ganglion. These preganglionic parasympathetics synapse in the submandibular ganglion and then provide secretomotor innervation to the submandibular and sublingual glands. The chorda tympani also contains taste fibers to the anterior 2/3 of the tongue, but not to the posterior 1/3.
The cell bodies of the taste fibers from the anterior two-thirds of the tongue are located in the:
Geniculate ganglion
Otic ganglion
Pterygopalatine ganglion
Submandibular ganglion
Trigeminal ganglion
The correct answer is: geniculate ganglion
The geniculate ganglion is the sensory ganglion of the facial nerve--it holds the cell bodies of the neurons that carry taste sensations from the anterior 2/3 of the tongue. These fibers then travel on the chorda tympani, which carries the fibers to the lingual nerve. The chorda tympani fibers then jump on the lingual nerve so they can reach the tongue and provide taste sensation to the anterior 2/3. The otic ganglion is the ganglion where parasympathetic fibers synapse before innervating the parotid gland. The lesser petrosal nerve is a branch of the glossopharyngeal nerve (CN IX) that carries presynaptic parasympathetic fibers to the otic ganglion. These fibers synapse in the otic ganglion, and the postsynaptic fibers travel on the auriculotemporal nerve to the parotid gland.

The pterygopalatine ganglion is the ganglion that enables secretomotor innervation to the mucous glands of the palate, nasal cavity, and lacrimal gland. Preganglionic fibers arrive at this ganglion from the greater petrosal nerve of the facial nerve. The submandibular ganglion receives preganglionic parasympathetic fibers from the chorda tympani; the postsynaptic fibers from the submandibular ganglion go to provide secretomotor innervation to the sublingual and submandibular glands. Finally, the trigeminal ganglia (also called the semilunar ganglia) is a sensory ganglion equivalent in histological structure and function to a dorsal root ganglion. The cell bodies of the afferent fibers of the trigeminal nerve reside in this ganglion.
Which of the following structures is located in the vestibule of the oral cavity?
Tongue
Opening of the parotid duct
Opening of the submandibular duct
Sublingual fold
Uvula
The correct answer is: the opening of the parotid duct
The oral vestibule is the space in the mouth lying between the lips and the teeth. See Netter Plate 45 for an illustration of the mouth and vestibule--although the vestibule isn't labeled, you can get some idea of where it lies. You can also see that the parotid duct is opening into this space . The parotid duct drains the parotid gland; it crosses the masseter to enter the cheek and drain into the oral cavity. The parotid duct drains into the cheek near the upper 2nd molar tooth.

The tongue is not in the vestibule--it's behind the teeth. The submandibular duct opens into the area under the tongue, near the tongue's frenulum. The sublingual fold is also underneath the tongue; this is the fold that contains the openings for the ducts of the sublingual gland to drain into the mouth. The uvula is the "punching bag" hanging from the soft palate at the back of the oral cavity.
When one presses the tongue tip against the anterior (incisor) teeth, which of the following muscles must contract?
Styloglossus
Hyoglossus
Genioglossus
Superior longitudinal
Verticalis
The correct answer is: genioglossus
Genioglossus is a large, fan shaped muscle with many actions: its inferior fibers protrude the tongue, its middle fibers depress the tongue, and its superior fibers draw the tip back and down. So, since the tongue is being protruded, the inferior fibers of genioglossus must be contracting. Styloglossus retracts and elevates the tongue. Hyoglossus depresses the sides of tongue and retracts the tongue. The superior longitudinal and verticalis muscles are intrinsic muscles of the tongue. They help shape the tongue for speech and chewing. All of these muscles are innervated by the hypoglossal nerve!
The muscle responsible for raising the floor of the mouth in the early stages of swallowing is the:
genioglossus
geniohyoid
hyoglossus
mylohyoid
palatoglossus
The correct answer is: mylohyoid
The mylohyoid muscle elevates the hyoid bone and the tongue and depresses the mandible. It is a muscle in the floor of the mouth that helps elevate the floor of the mouth, so this is the correct answer. The genioglossus is a large, fan shaped tongue muscle that protrudes tongue with its inferior fibers and depresses the tongue with its middle fibers. Geniohyoid is a small muscle in the floor of the mouth that elevates the hyoid and depresses the mandible. Hyoglossus is a tongue muscle that depresses the sides of the tongue and retracts the tongue. Finally, palatoglossus is a palate muscle, innervated by the vagus nerve. It elevates and retracts the tongue.
The cell bodies of the postganglionic parasympathetic neurons innervating the sublingual gland are found in which of the following ganglia?
ciliary
otic
submandibular
superior cervical
trigeminal
The correct answer is: submandibular ganglion
The submandibular ganglion contains the cell bodies of the postganglionic parasympathetic neurons that innervate the sublingual and submandibular gland. Preganglionic parasympathetic fibers arrived at the submandibular ganglion via the chorda tympani. These neurons then synapsed with the cell bodies in the submandibular ganglion, and the postganglionic fibers traveled out to provide secretomotor innervation to the sublingual and submandibular glands.

The ciliary ganglion receives preganglionic parasympathetic fibers from the inferior division of the oculomotor nerve. The postganglionic fibers from this ganglion leave via short ciliary nerves and innervate sphincter pupillae and the ciliary muscle of the eye. Also remember that sensory and sympathetic fibers are passing through the ciliary ganglion and distributing on the short ciliary nerves, but these fibers never synapse in that ganglion. The otic ganglion is the ganglion where parasympathetic fibers synapse before innervating the parotid gland. The lesser petrosal nerve is a branch of the glossopharyngeal nerve (CN IX) that carries presynaptic parasympathetic fibers to the otic ganglion. These fibers synapse in the otic ganglion, and the postsynaptic fibers travel on the auriculotemporal nerve to the parotid gland.

The superior cervical ganglia is at the level of the C1 and C2 vertebrae--it is the highest ganglia in the sympathetic chain. Sympathetic nerves from this ganglia go to form the internal carotid sympathetic plexus along the internal carotid artery. Finally, the trigeminal ganglia (also called the semilunar ganglia) a sensory ganglion equivalent in histological structure and function to a dorsal root ganglion. The cell bodies of the afferent fibers of the trigeminal nerve reside in this ganglion.
A 46-year-old female patient comes to the emergency department complaining of pain in the area just below her mandible on the right side of her face. She says that the pain is particularly severe when she eats. The area of the submandibular gland is tender and swollen, as is the area in the floor of her mouth lateral to the tongue. You suspect a stone in the submandibular duct, and a plain film radiograph shows a density in that region consistent with a stone. In order to remove the stone, the duct must be incised in the floor of the mouth. What nerve, that loops around the duct, is in danger in such an incision?
Chorda Tympani
Glossopharyngeal
Hypoglossal
Internal branch of the superior laryngeal
Lingual
The correct answer is: Lingual
The lingual nerve is found in the floor of the mouth--it wraps around the submandibular duct. So that nerve might be injured as you try to remove the stone from the submandibular duct. The lingual nerve contains general sensory fibers for the anterior 2/3 of the tongue, and fibers from the chorda tympani that provide taste to the anterior 2/3 of the tongue. All of these sensory components might be lost if the lingual nerve was damaged.

At the point where the lingual nerve is wrapping around the submandibular gland, the fibers from chorda tympani have already joined the lingual nerve.This means that fibers from chorda tympani might be damaged, but the chorda tympani itself is not near the submandibular duct. The glossopharyngeal nerve provides sensory and taste innervation to the posterior 1/3 of the tongue and sensory innervation to the pharynx--it is not associated with the submandbular duct. The hypoglossal nerve is in the sublingual space, just like the submandibular duct. However, it is deeper in the sublingual space and does not wrap around the duct. Remember--the lingual nerve is a much more superficial structure in the sublingual space. Finally, the internal branch of the superior laryngeal nerve crosses the thyrohyoid membrane to provide sensory innervation to the mucosa of the pharynx, superior to the vocal folds.
In accessing the submandibular gland in the submandibular triangle, what vessel coursing through the gland and triangle would need to be protected?
External jugular vein
Facial artery
Maxillary artery
Retromandibular vein
Superior thyroid artery
The correct answer is: facial artery
The submandibular triangle is formed by the lower border of the mandible and the anterior and posterior bellies of the digastric muscle. The facial artery and facial vein course through this triangle. The facial artery lies deep to the superficial part of the submandibular gland and wraps around the mandible. The facial vein is superficial to the gland--see Netter Plate 27 for a picture. The external jugular vein is a superficial vein on the lateral side of the neck. The maxillary artery is a branch of the external carotid artery that is the main source of blood to the infratemporal fossa. The retromandibular vein is a vein that passes through the parotid gland, along with the facial artery and facial nerve. Finally, the superior thyroid artery is a branch of the external carotid that supplies blood to the superior pole of the thyroid.
All of the following may be found in the paralingual space EXCEPT:
Hypoglossal nerve
Lingual nerve
Sublingual gland
Submandibular gland duct
Superficial lobe of the submandibular gland
The correct answer is: Superficial lobe of the submandibular gland
The paralingual space is found in the floor of the mouth, deep to the mylohyoid muscle. The space is bounded by the lateral tongue, the hyoid bone, and the oral mucosa. Remember--the submandibular gland has a deep and superficial portion. The deep portion of the submandibular gland is found deep to the mylohyoid muscle, while the superficial portion of the submandibular gland is found superficial to the mylohyoid muscle. So, the deep portion of the submandibular gland is in the paralingual space, while the superficial portion of the submandibular gland is not. See Netter Plate 55 for a picture of this.

The paralingual space also contains the submandibular duct and sublingual caruncle, the lingual nerve and submandibular ganglion, the hypoglossal nerve, and the sublingual gland and fold.
You are testing the extraocular muscles and their innervation in a patient who periodically experiences double vision. When you ask him to turn his right eye inward toward his nose and look downward he is able to look inward, but not down. Which nerve is most likely involved?
Abducens
Nasociliary
Oculomotor, inferior division
Oculomotor, superior division
Trochlear
The correct answer is: trochlear
To understand this question, you need to understand how the motions of the eye are tested. Since the actions of the extraocular muscles are complex, it is necessary to turn the eye to a position where a single action of each muscle predominates when evaluating the individual muscles. A key principle for muscle testing is: if a muscle has two actions and you perform one of those two, then it can't perform its other action. Superior and inferior recti turn the eye in and up or in and down. Superior and inferior oblique turn the eye out and down or out and up. So, if you turn your eye in (with the superior and inferior rectus as well as medial rectus), then only superior and inferior oblique can move the eye down or up (because the superior and inferior recti are already shortened by turning the eye in - they can't shorten any more). Similarly, if you turn the gaze out (with the obliques and lateral rectus) then only superior and inferior rectus can turn the eye up or down.

In this case, the patient has the eye turned inward, so the doctor must be testing the oblique muscles. The superior oblique muscle is the muscle that lowers the eye when it is turned inward. Since the patient can't do this, the superior oblique must not be functioning, and this muscle is innervated by the trochlear nerve.

Abducens (CN VI) innervates the lateral rectus muscle, which is not involved in the eye test. The nasociliary nerve comes from the ophthalmic division of the trigeminal nerve (V1). It is a sensory nerve to the eyeball that also carries some sympathetic fibers. The inferior division of the oculomotor nerve innervates inferior rectus, inferior oblique, and medial rectus. All of these muscles appear to be functioning. Finally, the superior division of the oculomotor nerve innervates levator palpebrae superioris and superior rectus. These are not the muscles that appear to be malfunctioning.
The outermost layer of the optic nerve sheath is a continuation of the:
Arachnoid membrane
Meningeal dura
Periosteal dura
Pia mater
Retina
The correct answer is: meningeal dura
The optic nerve comes off the base of the brain and passes through the optic canal. As it leaves the brain, it still retains all of the meningeal layer coverings. So, it is covered by meningeal dura, arachnoid membrane, and pia mater. This is significant, because an increase in intracranial pressure will increase the pressure in the subarachnoid space. This may squeeze the optic nerve and make the optic nerve bulge into the eye, a condition known as papilledema.

The periosteal dura is the layer of periosteum covering the internal surface of the calvaria. The retina is the inner layer of the eyeball which receives and absorbs visual light rays.
The inner lining of the eyelid is called the:
Orbital septum
Palpebral conjunctiva
Periorbita
Sclera
Tarsal plate
The correct answer is: palpebral conjunctiva
The palpebral conjunctiva is the thin membrane that lines the eyelid. It is continuous with the bulbar conjunctiva which lines the eyeball. The orbital septum is a weak membrane that spans from the tarsal plates to the margins of the orbit where it becomes continuous with the periosteum. It contains orbital fat and can limit the spread of infection in the orbit. The periorbita is the periosteum lining covering the bones forming the orbit. The sclera is the outer fibrous layer of the eyeball. Finally, the tarsal plate is a thin, cardboard-like layer of connective tissue in the eyelids which forms the "skeleton" of the eyelids.
What would the examining physician notice in the eye of a person who has taken a sympathetic blocking agent?
Exophthalmos and dilated iris
Enophthalmos and dry eye
Dry eye and inability to accommodate for reading
Wide open eyelids and loss of depth perception
Ptosis and miosis (pin-point pupil)
The correct answer is: Ptosis and miosis (pin-point iris)
Start this question out by thinking about what a sympathetic blocker would do to the pupil of the eye. Since sympathetic nerves allow the pupil to dilate, a sympathetic blocker would stop the eye from dilating and make the pupil constrict. Now think about the other issues. First, remember that sympathetic nerves innervate the superior tarsal muscle, which elevates the eyelids. If there is a problem with the regional sympathetics (as is the case with Horner's syndrome), the superior tarsal muscle will be paralyzed, and the eyelid will droop (ptosis). If the sympathetic nervous system is inhibited, sweating will cease, and you will observe the eye sinking back into the orbit.

Accomodation is not mediated by the sympathetic system; accomodation is a function of parasympathetic nerve so this should not be affected. Finally, the lacrimal gland is innervated by parasympathetics, so there should not be a major change in eye secretions after a sympathetic blocker. Putting all of these factors together, answer choice E is the only one that fits!
You are examining a patient who has a pituitary tumor involving the cavernous sinus. While doing a preliminary eye exam, you suspect the right abducens nerve of the patient has been damaged by the tumor. In which direction would you have the patient turn his right eye to confirm the defect?
Inward
Outward
Downward
Down and out
Down and in
Upward
Up and out
Up and in
The correct answer is: outward
To understand this question, you need to understand how the motions of the eye are tested. Since the actions of the extraocular muscles are complex, it is necessary to turn the eye to a position where a single action of each muscle predominates when evaluating the individual muscles. For the superior and inferior recti, turning the eye outward (abduction) by approximately 25 degrees places the superior rectus in position to raise the eye and the inferior rectus to lower the eye. Similarly, turning the eye inward (adduction) approximately 50 degrees places the inferior oblique in position to raise the eye and the superior oblique to lower the eye. The medial and lateral recti may be checked while the eye is staring straight ahead since they have simple planar actions.

In this case, you're interested in testing an "easy" muscle. Since the lesion appears to be in the abducens, which innervates the lateral rectus muscle, you could just ask the patient to turn the eye outward. If the patient could not do this, it would confirm that there was a lesion in the abducens nerve, since the muscle responsible for lateral movement of the eye would be paralyzed.

Also remember--a tumor in the cavernous sinus could affect many nerves. The oculomotor nerve (CN III), trochlear (CN IV), ophthalmic division of trigeminal (CN V1), and abducens (CN VI) all pass through the cavernous sinus.
You have a patient with a drooping right eyelid. You suspect Horner's syndrome. Which of the following signs on the right side would confirm this diagnosis?
Constricted pupil
Dry eye (lack of tears)
Exophthalmos
Pale, blanched face
Sweaty face
The correct answer is: constricted pupil
Horner's syndrome is a disorder involving damage to the sympathetic trunk in the neck. This means that the sympathetics of the head will be disrupted. This causes a variety of very characteristic symptoms, including a constricted pupil. Remember--sympathetic nerves innervate the dilator pupillae muscle. This muscle allows the eye to dilate. If these sympathetic nerves are lost, the pupil will contract.

Several of the other listed symptoms are the opposite of what you would expect with Horner's syndrome. Exophthalmos is the protrusion of the eye, but in Horner's syndome the eye sinks in, possibly due to the paralysis of a smooth muscle in the floor of the orbit. The face does not become blanched and sweaty with Horner's syndrome--instead, it becomes red and dry. Without the sympathetic nerve supply, the vasculature of the face cannot constrict. So, the arterioles in the patient's face are vasodilated, making the face red. Sympathetic nerves also innervate sweat glands; if these nerves are interrupted, the patient will not sweat and the face will appear very dry. Finally, the lacrimal gland is innervated by parasympathetics, not sympathetics. So, Horner's syndrome should produce no appreciable changes in tearing.

Make sure to know the different symptoms and signs of Horner's syndrome!
Following endarterectomy on the right common carotid, a patient is found to be blind in the right eye. It appears that a small thrombus embolized during surgery and lodged in the artery supplying the optic nerve. What artery would be blocked?
Central artery of the retina
Infraorbital
Lacrimal
Nasociliary
Supraorbital
The correct answer is: Central artery of the retina
The central artery of the retina is a branch of the ophthalmic artery. It is the sole blood supply to the retina; it has no significant collateral circulation and blockage of this vessel leads to blindness. The branches of this artery are what you view during a fundoscopic exam. The infraorbital artery is a branch of the maxillary artery. It comes through the infraorbital foramen, inferior to the eye. It supplies the maxillary sinus, the maxillary incisors, canine and premolar teeth, and the skin of the cheek below the orbit. The supraorbital artery is another branch of the ophthalmic artery. It comes through the supraorbital foramen or notch and supplies blood to the muscles, skin and fascia of the forehead. The lacrimal artery is a branch of the ophthalmic artery that supplies the lacrimal gland. The nasociliary artery doesn't exist, but there is a nasociliary nerve (the third and lowest branch of the ophthalmic division) that travels with the continuation of the ophthalmic artery.
You are asked to check the integrity of the trochlear nerve in the right eye of a patient. Starting with the eyes directed straight ahead, you would have the patient look:
Inward, toward the nose and downward
Inward, toward the nose and upward
Toward the nose in a horizontal plane
Laterally in a horizontal plane
Outward, away from the nose and downward
Outward, away from the nose and upward
The correct answer is: Inward, toward the nose and downward
To understand this question, you need to understand how the motions of the eye are tested. Since the actions of the extraocular muscles are complex, it is necessary to turn the eye to a position where a single action of each muscle predominates when evaluating the individual muscles. To test the superior and inferior recti, a patient needs to turn the eye outward approximately 25 degrees. At this postion, the superior rectus will simply act to raise the eye, and the inferior rectus will lower the eye. To test the superior and inferior obliques, a patient needs to turn the eye inward approximately 50 degrees. When the eye is in this position, the superior oblique muscle will act to lower the eye, and the inferior oblique will act to raise the eye.

So, now that you understand how to the test the eye, you have to decide which muscle is innervated by the trochlear nerve. And that's the superior oblique. So, to test this muscle, the eye needs to turn inward (toward the nose) and downward.

What nerves innervate the other muscles? The abducens nerve (CN VI) innervates the lateral rectus muscle. The oculomotor nerve (CN III) innervates the superior rectus, inferior rectus, medial rectus, and inferior oblique muscles.
The ducts of the lacrimal gland open into the:
Superior fornix of the conjunctiva
Inferior fornix of the conjunctiva
Lacrimal puncta
Lacrimal canaliculi
Lacrimal lake
The correct answer is: Superior fornix of the conjunctiva
Lacrimal fluid is produced by the lacrimal gland, which lies in a fossa in the superolateral part of each orbit. The fluid from this gland enters the conjunctival sac through up to 12 lacrimal ducts that open into the superior conjunctival fornix. The tears then flow to the medial angle of the eye and collect in the lacrimal lake. The lacrimal papilla are small elevations on the eyelids, found near the lacrimal lake. These papillae have small openings called the lacrimal puncta; tears flow from the lacrimal lake into these puncta. From there, the lacrimal fluid goes into small canniliculi which drain the fluid into the lacrimal sac. The lacrimal sac continues on as the nasolacrimal duct and drains tears into the inferior nasal meatus. Take a look at Netter Plate 77 and try to follow the path of tears from the lacrimal gland to the inferior meatus!
Starting from a position gazing straight ahead, to direct the gaze downward, the inferior rectus muscle must be active along with the:
Superior oblique
Inferior oblique
Medial rectus
Lateral rectus
Superior rectus
The correct answer is: Superior Oblique
The inferior rectus muscle depresses the eye and medially rotates it. So, to direct the gaze downward, you want to find a muscle that will depress the eye while counterbalancing the medial rotation with lateral rotation. And, the superior oblique, innervated by the trochlear nerve (CN IV), does just that--it depresses the eye while laterally rotating it. The inferior oblique muscle laterally rotates the eye and elevates the eye. The medial rectus adducts the eye--it does not raise or lower the eye. The lateral rectus abducts the eye--it also does not raise or lower the eye. Finally, the superior rectus elevates the eye and draws it medially.
During a physical examination it is noted that a patient has ptosis. What muscle must be paralyzed?
Orbicularis oculi, lacrimal part
Orbicularis oculi, palpebral part
Stapedius
Superior oblique
Superior tarsal (smooth muscle portion of levator palpebrae)
The correct answer is: Superior tarsal
The superior tarsal muscle is a smooth muscle which is sympathetically innervated. It is an involuntary muscle that elevates the eyelid. It is innervated by the cervical sympathetic trunk, and this muscle's functioning provides a good indication of the integrity of the cervical sympathetic trunk. If the cervical sympathetic trunk has been damaged, a patient will have ptosis, a droopy eyelid. Orbicularis oculi is innervated by the facial nerve. If this muscle is paralyzed, the problem won't be a droopy eyelid--instead, the patient won't be able to close the eyelid. This is why patients with Bell's palsy are prescribed lubricating eye drops--if they can't close the eyelid, they may be at risk for corneal irritation. Stapedius is another muscle innervated by the facial nerve -- it serves to dampen the vibrations of the stapes and the tympanic membrane. Finally, the superior oblique muscle depresses the eyeball and turns it laterally. It does not affect the eyelid.
The extraocular muscle that does not originate at or near the apex of the orbit is the :
Inferior oblique
Inferior rectus
Levator palpebrae superioris
Superior oblique
Superior rectus
The correct answer is: Inferior oblique
The inferior oblique muscle does not originate at the apex of the orbit. It takes origin from the floor of the orbit, lateral to the lacrimal groove. The inferior rectus and superior rectus muscles take origin from the common tendinous ring at the apex of the orbit. The levator palpebrae superioris takes origin from the apex of the orbit above the optic canal. The superior oblique muscle takes origin from the apex of the orbit, above the optic canal. For a picture of this, see Netter Plate 79.
An adolescent boy suffers from severe acne. As is often the case he frequently squeezed the pimples on his face. He subsequently develops a fever and deteriorates into a confused mental state and drowsiness. He is taken to his physician and after several tests a diagnosis of cavernous sinus infection and thrombosis is made. The route of entry to the cavernous sinus from the face was most likely the:
Carotid artery
Mastoid emissary vein
Middle meningeal artery
Ophthalmic vein
Parietal emissary vein
The correct answer is: Ophthalmic vein
The ophthalmic veins are continuous with the facial vein and the pterygoid plexus of veins. These veins drain the face toward the cavernous sinus. They are valveless, so infections from the face can drain into the cavernous sinus. Besides causing fever and confusion, thrombotic congestion and edema in the cavernous sinus can compress the nerves that traverse that space to exit through the superior orbital fissure(CN III, CN IV, CN V1, and CN VI). This can affect the function of the ocular muscles, so one symptom of a cavernous sinus infection might be an inability to perform different eye movements.

The carotid artery and middle meningeal artery would not be the source of the infections. Infections do not tend to enter through arterial circulation. Remember--the common carotid is the major source of blood to the head and neck, and the middle meningeal artery is the branch of the maxillary artery that supplies blood to the dura. The emissary veins are valveless veins of the scalp. These veins can carry blood from the scalp to the dural venous sinuses or in the reverse direction depending on blood pressure. These veins may carry infectious materials from the scalp into the dural venous sinuses, but they are not important for carrying infections to the cavernous sinus.
If a person looking inward towards their nose is unable to look down, which nerve may be injured?
Abducens (CN VI)
Inferior division of oculomotor (III)
Optic (II)
Superior division of oculomotor (III)
Trochlear (IV)
The correct answer is: Trochlear (IV)
To understand this question, you need to understand how to evaluate the muscles of the eye. Since the actions of the extraocular muscles are complex, it is necessary to turn the eye to a position where a single action of each muscle predominates. To isolate the superior and inferior recti, the patient needs to turn the eye outward by approximately 25 degrees. This places the superior rectus in position to raise the eye and the inferior rectus in position to lower the eye. Turning the eye inward approximately 50 degrees places the inferior oblique in position to raise the eye and the superior oblique in position to lower the eye. The medial and lateral recti are the easy muscles -- they may be checked while the eye is staring straight ahead since they have simple planar actions

So, this patient is looking inward, which means that the obliques are being tested. The patient can't look downward, which shows that the superior oblique is not functional. This is the only muscle innervated by the trochlear nerve (CN IV).

Abducens (CN VI) innervates the lateral rectus muscle, which is tested by asking the patient to move the eye outward. The inferior division of the oculomotor nerve innervates inferior rectus, inferior oblique, and medial rectus. The superior branch of the oculomotor nerve innervates levator palpebrae superioris and superior rectus muscles. Finally, the optic nerve (CN II) provides the special sense of vision, and it is not tested in the eye-movement tests.

Are you getting the idea that you really need to know about testing the eye muscles? Take the time and really understand this concept--you'll be glad that you did!
If a person is taking a sympathetic blocking agent, what would you notice in her or his eyes?
Dry eyes and inability to accommodate for reading
Enophthalmos and teary eyes (III)
Exophthalmos and dilated pupil
Ptosis and constricted pupil
Wide open eyes and loss of depth perception (IV)
The correct answer is: Ptosis and constricted pupil
To understand this question, it's important to look at all the different choices and determine which ones fit with a sympathetic block. First, the lacrimal gland is innervated parasympathetically, so a sympathetic blocker should have no effect on eye secretions. Accomodation is also a function of the parasympathetic nervous system; it should not be altered by a sympathetic blocker. Enophthalmos is the name for the eye sinking into its orbit. A sympathetic block does cause enophthalmos, due to the paralysis of a smooth muscle in the floor of the orbit. Exophthalmos is the opposite of enophthalmos--it is the protrusion of the eye from the orbit. You would not see exophthalmos with a sympathetic blockade. Sympathetic nerves allow the eye to dilate--if you blocked these nerves, the eye would constrict. A sympathetic blocker would also cause ptosis--it would paralyze the superior tarsal muscle, which holds the lids up involuntarily and receives sympathetic innervation. Finally, the sympathetic blocker should not affect depth perception. If you put all of these things together, answer choice D is the correct one.

If it helps to remember, taking a sympathetic blocking agent will lead to similar symptoms in the head and neck as Horner's syndrome, a disease characterized by a loss of sympathetic innervation to the head and neck.
The entry of bacteria through which space could lead to an infection in the mastoid air cells:
Auditory (nasopharyngeal) tube
Cochlea
External acoustic meatus
Internal acoustic meatus
Sacculus
The correct answer is: Auditory (nasopharyngeal) tube
The auditory (nasopharyngeal) tube is a connection between the nasal portion of the pharynx and the tympanic cavity that allows pressure to equalize on either side of the tympanic membrane. It is located in the anterior wall of the middle ear and is comprised of bone at the tympanic end and cartilage on the pharyngeal end. The pharyngeal mucosa is continuous with the lining of the tympanic cavity and mastoid air cells. This allows infectious material to pass to the middle ear and mastoid area.

The cochlea is the organ of hearing that receives, interprets, and transmits sound via the vestibulocochlear nerve (CN VIII). The external acoustic meatus is the opening in the temporal bone that allows sound waves to reach the tympanic membrane. The internal acoustic meatus is the foramen in the temporal bone that allows the vestibulocochlear nerve and the facial nerve to pass into the skull at the base of the brain. Finally, the sacculus is a fluid filled sac that is part of the balancing apparatus of the ear - it is located in the vestibule of the ear.
Which structure is attached to the center of the tympanic membrane?
Foot plate of the stapes
Handle (manubrium) of the malleus
Long process of the incus
Tragus
Utricle
The correct answer is: handle (manubrium) of the malleus
The handle of the malleus is attached to the center of the tympanic membrane. The head of the malleus then articulates with the body of incus, and the long process of incus articulates with the head of stapes. The foot plate of stapes sits in the oval window. Take a look at Netter Plate 88 for a better idea of how these ossicles connect!

The tragus is a cartilagenous structure of the external ear, located anterior to the external auditory meatus. It is hard to describe the tragus; it is labeled in the upper left picture of Netter Plate 88. The utricle is a part of the balancing apparatus of the ear--like the saccule, it is a fluid filled sac in the vestibule.
A 45-year old woman with recurrent left middle ear infection (otitis media) complained of partial dryness of her mouth to her ENT surgeon. Taste sensation and hearing were normal. After a thorough clinical examination at the hospital, the doctor concluded that the infection must have spread to a component of the glossopharyngeal nerve (CN IX) that supplies the parotid gland. On which of the following walls of the middle ear is this nerve component located?
Anterior
Lateral
Medial
Posterior
Roof
The correct answer is: Medial
To answer this question, you first need to identify the component of the glossopharyngeal nerve that innervates parotid gland. Remember--preganglionic parasympathetic fibers from CN IX are arriving at the otic ganglion via the lesser petrosal nerve and synapsing in the ganglion; the postganglionic fibers then travel on the auriculotemporal nerve (V3) and innervate the parotid gland. So, the lesser petrosal nerve, or fibers that create the lesser petrosal nerve, must have been injured. Now, you just need to think about where these fibers are in the ear.

These fibers from the glossopharyngeal nerve are covering the promontory on the medial wall of the ear. The tympanic nerve is a branch of the glossopharyngeal nerve that carries the preganglionic parasympathetic fibers that will eventually travel to the otic ganglion. The tympanic nerve lies on the promontory and creates the tympanic plexus, which gives rise to the lesser petrosal nerve. Given the clinical presentation, the patient must have an infection in the tympanic nerve, tympanic plexus or lesser petrosal nerve. And, all of these nerves are associated with the medial wall of the middle ear.

What are the important associations with the other walls of the ear? The lateral wall is formed by the tympanic membrane, and the chorda tympani courses across this membrane. The anterior wall contains the opening of the auditory tube and a semicanal for tensor tympani. The posterior wall of the ear has the aditus ad antrum, which is the entrance to the mastoid antrum and air cells. The facial nerve (VII) and a small branch of the facial nerve which innervates stapedius are found on the posterior wall.
A 3-year-old girl ruptured her eardrum when she inserted a pencil into her ear. Her mother took her to the emergency department after noticing that the child was crying and complaining of pain in her ear with a few drops of blood in the external auditory meatus. The attending doctor examined the child for possible injury to a nerve that runs across the eardrum. The most likely nerve to be injured is the:
Auricular branch of the vagus
Chorda tympani
Glossopharyngeal (CN IX)
Lesser petrosal
Trigeminal (CN V)
The correct answer is: chorda tympani
When this girl ruptured her eardrum, she damaged the tympanic membrane. So, you need to think about the nerve and structures that are associated with the tympanic membrane (ie, on the lateral wall of the middle ear). Chorda tympani lies across the tympanic membrane, so it's possible that this nerve was injured by the pencil.

The auricular branch of the vagus nerve is a small branch of the vagus that supplies afferent sensory innervation to the external acoustic meatus. This nerve is not close to the tympanic membrane The glossopharyngeal nerve and lesser petrosal nerve are associated with the promontory of the ear, which is on the medial wall of the middle ear. The trigeminal nerve is not close to the ear and would not be damaged by the injury.

What might happen if chorda tympani was injured? No taste sensation to the anterior 2/3 of the tongue and no secretomotor innervation to the sublingual and submandibular glands!
An elderly patient with chronic otitis media (middle ear infection) might have all the following complications EXCEPT:
Inabilty to chew food due to injury to the mandibular division of the trigeminal nerve (CN V)
Loss of taste in the anterior part of the tongue due to injury to the chorda tympani nerve
Mastoiditis
Paralysis of facial muscles due an injury to the facial nerve (CN VII)
Some degree of deafness due to damage to the ossicles
The correct answer is: Inability to chew food due to injury to the mandibular division of the trigeminal nerve (CN V3)
The mandibular division of the trigeminal nerve is not associated with the middle ear. So, chronic ear infections should have no effect on this structure. All of the other answers refer to structures that are closely associated with the middle ear--a chronic infection in this area could cause any of the other listed complications. The chorda tympani nerve travels along the lateral wall of the middle ear, running across the tympanic membrane. It could be damaged by chronic infection and inflammation. Remember--chorda tympani provides secretomotor innervation to the submandibular and sublingual glands and taste sensation to the anterior 2/3 of the tongue. Mastoiditis is an infection of the mastoid air cells. Since these air cells connect to the middle ear through the aditus ad antrum on the posterior wall, an infection in the middle ear could easily spread to the mastoid air cells. The facial nerve is also located on the posterior wall of the middle ear, so it could also be damaged by the chronic infection. Finally, a chronic infection can damage the three ossicles and lead to deafness.
A patient with a facial nerve paralysis suffers from inability to dampen loud noises (hyperacusis) due to denervation of which muscle?
Posterior belly of digastric
Stapedius
Tensor tympani
Stylohyoid muscle
The correct answer is: Stapedius
Stapedius is a small muscle in the ear innervated by the facial nerve. It dampens large vibrations of the stapes and the tympanic membrane; this allows the ear to diminish loud noises. If the facial nerve is paralyzed (as seen with Bell's palsy), the nerve to stapedius is lost, and the ear cannot lessen the vibrations of stapedius. This causes hyperacusis.

The digastric muscle elevates the hyoid bone and depresses the mandible. Its posterior belly is innervated by the facial nerve, so this muscle would be paralyzed if the facial nerve was damaged. However, the posterior belly of the digastric is not involved with the ear. Tensor tympani is a muscle in the ear with a similar function to stapedius--it dampens vibrations of the tympanic membrane. This muscle is innervated by the mandibular branch of the trigeminal nerve (V3)--not the facial nerve. Stylohyoid elevates and retracts the hyoid bone. It's innervated by the facial nerve, but it does not have any effect on the ear.
All of the following are true about the middle ear EXCEPT:
The joints between ossicles are synovial
The chorda tympani nerve is related to the lateral wall
The facial nerve passes in a canal situated in the medial and anterior walls
The auditory tube connects the nasopharynx with the anterior wall
Its mucous membrane is supplied by the glossopharyngeal nerve (CN IX)
The correct answer is: The facial nerve passes in a canal situated in the medial and anterior wall.
The facial nerve passes in a canal situated in the posterior and medial walls of the middle ear. It is not associated with the anterior wall. The other 4 statements are true. The ossicles articulate with each other at synovial joints. The chorda tympani is a branch of the facial nerve that is related to the lateral wall of the middle ear, near the tympanic membrane. It passes between the malleus and incus. The auditory tube is found in the anterior wall--it is a path of communication between the nasal portion of the pharynx and the tympanic cavity that allows pressure to equalize on either side of the tympanic membrane. Finally, the mucous membrane of the middle ear is supplied by branches of the tympanic plexus, which is formed by the tympanic branch of the glossopharyngeal nerve.
A patient has sustained a fracture to the base of the skull. Thorough examination concluded that the right greater petrosal nerve, among other structures, has been injured. This conclusion was based on which of the patient's signs:
Partial dryness of the mouth due to lack of salivary secretions from the submandibular and sublingual glands
Partial dryness of the mouth due to lack of salivary secretions from the parotid gland
Dryness of the right cornea due to lack of lacrimal secretion
Loss of taste sensation from the right anterior 2/3rd of the tongue
Loss of general sensation from the right anterior 2/3rd of the tongue
The correct answer is: Dryness of the right cornea due to lack of lacrimal secretion.
The greater petrosal nerve is a branch of the facial nerve that arises adjacent to the geniculate ganglion. It carries preganglionic parasympathetic fibers to the pterygopalatine ganglion. The fibers synapse in this ganglion, and the postsynaptic parasympathetic fibers go on to innervate the lacrimal gland, mucous glands of the palate, and the mucous glands of the nasal cavity. So, if the greater petrosal nerve was damaged, the presynaptic parasympathetic fibers would be lost, and the lacrimal gland would not function properly.

The sublingual and submandibular glands are innervated by the chorda tympani, which brings preganglionic parasympathetic fibers to the submandibular ganglion. The chorda tympani also provides taste sensation to the anterior 2/3 of the tongue. Although the chorda tympani, like the greater petrosal nerve, is a branch of the facial nerve, the chorda tympani never runs with the greater petrosal nerve. So, injuring the greater petrosal nerve would not harm the chorda tympani. The parotid gland is innervated by the lesser petrosal nerve, a branch of the glossopharyngeal nerve (CN IX). The fibers from this nerve go to the otic ganglion, synapse there, and then continue on to the parotid gland via the auriculotemporal nerve. Finally, general sensation to the anterior 2/3 of the tongue is carried by the lingual nerve, a branch of V3.
The location of the otic ganglion is in the:
Pterygopalatine fossa
Internal ear
Infratemporal fossa
Middle cranial fossa
None of the above
The correct answer is: Infratemporal fossa
The otic ganglion is in the infratemporal fossa, just inferior to the foramen ovale, medial to the mandibular nerve and posterior to the medial pterygoid muscle. It is the ganglion where fibers from the lesser petrosal nerve synape. The postsynaptic parasympathetic fibers from the otic ganglion are secretory to the parotid gland--they reach the parotid gland by the auriculotemporal nerve. The pterygopalatine fossa is a small pyramidal space inferior to the apex of the orbit. It lies between the pterygoid process of the sphenoid bone posteriorly and the posterior aspect of the maxilla anteriorly. It contains the terminal part of the maxillary artery, the maxillary nerve, and the pterygopalatine ganglion. The internal ear is buried in the petrous part of the temporal bone--it contains the vestibulocochlear organ which allows for the reception of sound and maintaining balance.

Finally, the middle cranial fossa is the large depression in the cranial base, formed by the greater wings of the sphenoid and squamous parts of the temporal bones laterally and the petrous parts of the temporal bones posteriorly. It contains four important foramina: the superior orbital fissure, which transmits the ophthalmic veins and the nerves entering the orbit (CN III, CN IV, CN V1 and CN VI); the foramen rotundum which transmits CN V2; the foramen ovale, which transmits CN V3, and the foramen spinosum, which transmits the middle meningeal vessels.
A patient complains of loss of hearing in the right ear. Examination reveals ankylosis (otosclerosis) of the footplate of the stapes to the surrounding bone. Which part of the bony labyrinth is involved?
Aditus ad antrum
Cochlear (round) window
Cochlear duct
Internal acoustic meatus
Vestibular (oval) window
The correct answer is: Vestibular (Oval) Window
The oval window is a fossa in the medial wall of the middle ear--it is found just above the promontory. It leads into the vestibule of the bony labyrinth. In life, this fossa is completely covered by the footplate of the stapes. So, if there was otosclerosis of the stapes to the surrounding bone, there would be damage to the oval window. The round window is also a fossa in the medial wall of the middle ear, but it is located below the promontory. The round window is covered by a thin membrane.

The aditus ad antrum is an opening on the posterior wall of the middle ear. It is the opening to the mastoid antrum and mastoid air cells. The cochlear duct is the spiral tube suspended in the cochlear canal--it is part of the membranous labyrinth. Finally, the internal acoustic meatus is a foramen in the base of the skull. The facial nerve (CN VII) and the vestibulocochlear nerve (CN VIII) enter the petrous temporal bone through this foramen.
The geniculate ganglion is the sensory ganglion of which nerve:
Facial
Glossopharyngeal
Trigeminal
Vagus
Vestibulocochlear
The correct answer is: Facial nerve
The geniculate ganglion is the sensory ganglion of the facial nerve. These fibers travel as part of the chorda tympani to provide taste sensation to the anterior 2/3 of the tongue. The glossopharyngeal nerve has superior and inferior ganglia, located near the jugular foramen. These ganglia contain the cell bodies for the afferent components of this nerve. The trigeminal nerve has a trigeminal ganglion which contains the cell bodies of neurons composing the sensory root of this nerve. The vagus nerve also has a superior and inferior ganglion, which are located near the ganglia of the glossopharyngeal nerve. The superior ganglion is concerned with the general sensory component of the nerve, and the inferior ganglion is concerned with the visceral sensory component of the nerve. Finally, the vestibulocochlear nerve has a vestibular ganglion, which houses the neurons for balance, and the cochlear ganglion, which houses the neurons concerned with hearing.
Repeated middle ear infections have destroyed the tympanic plexus in the middle ear cavity. The loss of preganglionic parasympathetic fibers that pass through the plexus diminish production of:
Mucus in the nasal cavity
Mucus on the soft palate
Saliva by the parotid gland
Saliva by the submandibular and sublingual glands
Tears by the lacrimal gland
The correct answer is: Saliva by the parotid gland
The tympanic plexus is formed by the tympanic nerve, a branch of the glossopharyngeal nerve (CN IX). The tympanic plexus provides sensory innervation to the mucosal lining of the middle ear. The lesser petrosal nerve also comes out of the tympanic plexus. This nerve carries preganglionic parasympathetic fibers from the tympanic plexus - these fibers eventually synapse in the otic ganglion. The postsynaptic fibers that leave the otic ganglion provide parasympathetic secretomotor innervation to the parotid gland. So, if the tympanic plexus was destroyed, the lesser petrosal nerve would be destroyed and the parotid gland would not secrete saliva.

Mucosal secretions in the nasal cavity and the soft palate and tear secretions by the lacrimal gland are all mediated by the postganglionic fibers of the pterygopalatine ganglion. This ganglion receives preganglionic fibers from the greater petrosal nerve, a branch of the facial nerve (CN VII). The submandibular and sublingual glands receive their preganglionic fibers from the chorda tympani - another branch of the facial nerve.
The soft palate is active in all of the following except:
Breathing
Chewing
Coughing
Swallowing
Yawning
The correct answer is: Breathing
The soft palate is the movable posterior 1/3 of the palate, which is suspended from the posterior border of the hard palate. When a person swallows, the soft palate is initially tensed to allow the tongue to press against it, squeezing the bolus of food to the back of the mouth. The soft palate is elevated posteriorly and superiorly against the wall of the pharynx, thereby preventing the passage of food into the nasal cavity. The soft palate functions similarly to prevent the bolus of food from passing into the nasal cavity while chewing and to prevent expectorated material from entering the nasal cavity while coughing. The soft palate also elevates when yawning. The soft palate does not elevate during breathing--if it did rise during breathing, the air inspired through the nose might be blocked from entering the trachea.
Most paranasal sinuses and/or air cells drain, directly or indirectly, into the:
Inferior meatus
Middle meatus
Superior meatus
Nasal vestibule
Sphenoethmoidal recess
The correct answer is: middle meatus
The middle meatus contains the semilunar hiatus, which receives drainage from the frontonasal duct (draining the frontal sinus), the anterior ethmoidal air cells, and the maxillary sinus. So, it's draining most of the paranasal sinuses. The inferior meatus receives the nasolacrimal duct which conducts tears from the orbit. The superior meatus receives drainage from the posterior ethmoidal air cells. The nasal vestibule is the opening of the nose--none of the sinuses drain directly into this area. The sphenoethmoidal recess is the opening for the sphenoethmoidal sinus.
Irrigation of the maxillary sinus through its opening is a supportive measure to accelerate the resolution of a maxillary sinus infection. Which of the following nasal spaces is the most likely approach to the sinus opening?
choana
inferior meatus
middle meatus
sphenoethmoidal recess
superior meatus
The correct answer is: middle meatus
The middle meatus contains the semilunar hiatus, which receives drainage from the maxillary sinus, the frontonasal duct (draining the frontal sinus), and the anterior ethmoidal air cells. The maxillary sinus is draining into the middle meatus, so it would be easiest to approach the maxillary sinus through this space. The choana is the space where the nasal cavity opens into the nasopharynx--it is found at the very posterior border of the nasal cavity. The inferior meatus receives the nasolacrimal duct which conducts tears from the orbit. The sphenoethmoidal recess is the opening for the sphenoethmoidal sinus. Finally, the superior meatus receives drainage from the posterior ethmoidal air cells.
In assessing a deep laceration of the right side of the nose, the attending physician determines that the cartilage on the lateral side of the nostril has been cut. What cartilage was injured?
Accessory
Alar
Lateral
Septal
None of the above
The correct answer is: alar cartilage
The alar cartilage is found on the inferolateral side of the nose. This is probably the cartilage that was injured in this laceration. The septal cartilage creates the septum of the nose. The lateral cartilage is a specialization of the septal cartilage that is superior to the alar cartilage, also on the lateral side of the nose. The accessory cartilage is a small piece of cartilage between the alar cartilage and the lateral cartilage
The nasolacrimal duct empties into which part of the nasal cavity?
Hiatus semilunaris
Inferior meatus
Middle meatus
Sphenoethmoidal recess
Vestibule
The correct answer is: inferior meatus
The inferior meatus receives the nasolacrimal duct which conducts tears from the orbit. The semilunar hiatus is found in the middle meatus--it receives drainage from the frontonasal duct (draining the frontal sinus), the anterior ethmoidal air cells, and the maxillary sinus. The sphenoethmoid recess is the opening for the sphenoethmoidal sinus. The vestibule of the nose is the opening of the nose that is covered with skin and stiff hairs.
The pharyngeal tonsils, or adenoids, may become inflamed and in serious cases need to be removed. Where would the physician search for them?
In the tonsillar fossa
In the pharyngeal recess
In the piriform recess
In the roof of the nasopharynx
Upon the dorsal surface of the tongue
The correct answer is: In the roof of the nasopharynx
The pharyngeal tonsil is located on the roof of the nasopharynx. The pharyngeal recess is a space located posterior to the torus tubarius in nasopharynx. The tonsillar fossa is the space where the palatine tonsil is located--it is found between the palatoglossal fold and the palatopharyngeal fold. The piriform recess is a shallow depression located lateral to the aryepiglottic fold in the laryngopharynx. This is a common place where food can get stuck. The piriform recess is also significant because the internal branch of the superior laryngeal nerve is located immediately deep to the mucosa of this region--so, an injury here may damage this nerve! Finally, the lingual tonsil is found on the dorsal surface of the tongue.
The middle nasal concha is part of what bone?
Ethmoid bone
Maxilla
Palatine bone
Sphenoid bone
Vomer
The correct answer is: ethmoid
The superior and middle nasal conchae are part of the ethmoid bone. The maxillary bone is the bone that forms the midface. It forms the inferior orbital margin and contains the teeth and maxillary sinus. The palatine bone forms the posterior part of the hard palate. The sphenoid bone is an irregularly shaped bone forming the central portion of the skull. It has many parts, including a body, greater wing, lesser wing and pterygoid plates. The vomer is a thin plate of bone forming the posteroinferior part of the nasal septum.
A 23-year-old man comes to you complaining that he can't stop crying, i.e. tears regularly run down the right side of his face. You suspect that one of the lacrimal ducts on the right side of the face is blocked. You look into an endoscope to see if the nasolacrimal duct is blocked. Into which part of the nasal cavity would you look to see the opening of the duct?
Hiatus semilunaris
Inferior meatus
Middle meatus
Sphenoethmoidal recess
Superior meatus
The correct answer is: Inferior meatus
The inferior meatus receives the nasolacrimal duct which conducts tears from the orbit. So, this is the place where you should be looking to see the opening of the nasolacrimal duct. The semilunar hiatus is found in the middle meatus--it receives drainage from the frontonasal duct (draining the frontal sinus), the anterior ethmoidal air cells, and the maxillary sinus. The sphenoethmoid recess is the opening for the sphenoethmoidal sinus. The superior meatus is the opening for the posterior ethmoidal air cells.
Which structure forms a border of the tonsillar fossa?
Torus tubarius
Palatopharyngeal fold
Salpingopharyngeal fold
Lateral glossoepiglottic fold
Medial pterygoid plate
The correct answer is: Palatopharyngeal fold
The tonsillar fossa is the place where you find the palatine tonsil - it is a space found between the palatoglossal and palatopharyngeal folds. Torus tubaris is a mucosal fold covering the anteromedial end of the auditory tube cartilage - it projects toward the midline from the lateral wall of the nasopharynx. The salpingopharyngeal fold is a vertical fold of mucous membrane that extends from the medial end of the auditory tube - it covers the salpingopharyngeus muscle. The lateral glossoepiglottic fold is a small fold extending from the tongue to the epiglottis - it is found posterior to the tonsillar fossa. Finally, the medial pterygoid plate is a part of the sphenoid bone in the skull - it is the attachment of the superior pharyngeal constrictor muscle and the pharyngobasilar fascia.
The communication between the pharynx and the nasal cavity is known as the:
Aditus
Auditory tube
Choanae
Fauces
Piriform recess
The correct answer is: choanae
The choanae are the opening at the posterior border of the nasal cavity that allow the nasal cavity to communicate with the nasopharynx. The aditus is the laryngeal inlet - this is the space that is covered by the epiglottis when swallowing. The auditory tube is a tube that connects the nasopharynx with the middle ear, allowing for pressure to equalize on both sides of the tympanic membrane. The fauces is the passage from the mouth to the oropharynx, including the lumen and its boundaries. Finally, the piriform recess is a shallow depression located lateral to the aryepiglottic fold in the laryngopharynx. This is a place where food is commonly lodged.